Anda di halaman 1dari 98

ICD-10 CM and ICD-10 PCS

Workbook

-Proprietary and Confidential-


©2014 himagine solutions inc.| 1001 East Palm Avenue | Tampa, FL 33605 | 866-230-7865 | ICD10@himaginesolutions.com
Table of Contents
Introduction to ICD-10...................................................................................................................................................6
ICD-10-CM System ...........................................................................................................................................................9
Benefits of ICD-10-CM................................................................................................................................................9
Similarities and Differences between the two Coding Systems....................................................................9
ICD-9-CM Diagnoses Codes: .................................................................................................................................. 10
ICD-10-CM Diagnosis Codes:................................................................................................................................. 10
New features found in ICD-10-CM....................................................................................................................... 10
Structural Changes in ICD-10-CM:....................................................................................................................... 10
ICD-10-CM Guidelines ................................................................................................................................................. 28
ICD-10-CM Coding Guideline changes and additions .................................................................................... 28
Organization and Classification of Diseases and Disorders – Chapters 1–7 .............................................. 32
Chapter 1: Certain infectious and parasitic diseases (A00-B99)............................................................... 32
Chapter 2: Neoplasms (C00-D49)........................................................................................................................ 34
Chapter 3: Diseases of the blood and blood-forming organs and certain disorders involving the
immune mechanism (D50-D89) .......................................................................................................................... 36
Chapter 4: Endocrine, nutritional and metabolic diseases (E00-E89) .................................................... 37
Chapter 5: Mental and behavioral disorders (F01-F99) .............................................................................. 39
Chapter 6: Diseases of the nervous system (G00-G99) ................................................................................ 41
Chapter 7: Diseases of eyes and adnexa (H00-H59)...................................................................................... 42
Organization and Classification of Diseases and Disorders – Chapters 8–14............................................ 45
Chapter 8: Diseases of the ear and mastoid process (H60-H95)............................................................... 45
Chapter 9: Diseases of the circulatory system (I00-I99) ............................................................................. 46
Chapter 10: Diseases of the respiratory system (J00-J99) .......................................................................... 48
Chapter 11: Diseases of the digestive system (K00-K94)............................................................................ 50
Chapter 12: Diseases of skin and subcutaneous tissue (L00-L99) ........................................................... 52
Chapter 13: Diseases of the musculoskeletal system and connective tissue (M00 -M99) ................. 54
Chapter 14: Diseases of the genitourinary system (N00-N99).................................................................. 56
Organization and Classification of Diseases and Disorders – Chapters 15–21 ......................................... 60
Chapter 15: Pregnancy, childbirth and the puerperium (O00-O9A)........................................................ 60
Chapter 16: Certain conditions origination in the perinatal period (P00-P96) ................................... 62
Chapter 17: Congenital malformations, deformations and chromosomal abnormalities (Q00 -Q99)
........................................................................................................................................................................................ 64
Chapter 18: Symptoms, signs and abnormal clinical and laboratory findings, not elsewhere
classified (R00-R99) ................................................................................................................................................ 65
Chapter 19: Injury, poisoning and certain other consequences of external causes (S00-T88)....... 67

-Proprietary and Confidential-


©2014 himagine solutions inc.| 1001 East Palm Avenue | Tampa, FL 33605 | 866-230-7865 | ICD10@himaginesolutions.com
Chapter 20: External causes of morbidity (V00-Y99)................................................................................... 71
Chapter 21: Factors influencing health status and contact with health services (Z00-Z99)............ 74
Overview of ICD-10-CM Classification Changes by Body System/Chapter (Not all Inclusive) ............ 78
Circulatory .................................................................................................................................................................. 78
Respiratory................................................................................................................................................................. 78
Nervous System......................................................................................................................................................... 79
Endocrine System ..................................................................................................................................................... 80
Blood and Blood-Forming Organs....................................................................................................................... 81
Infectious Diseases................................................................................................................................................... 82
OB-GYN ........................................................................................................................................................................ 83
Urinary System .......................................................................................................................................................... 84
ICD-10-PCS System ..............................................................................................Error! Bookmark not defined.
Overview.............................................................................................................Error! Bookmark not defined.
Characters ..........................................................................................................Error! Bookmark not defined.
Components of ICD-10-PCS and Table Organization ...........................Error! Bookmark not defined.
Definitions Used in ICD-10-PCS...................................................................Error! Bookmark not defined.
ICD-10 PCS Official Guidelines and characteristics ...................................Error! Bookmark not defined.
ICD-10 PCS Characteristics...........................................................................Error! Bookmark not defined.
ICD-10 Conventions ........................................................................................Error! Bookmark not defined.
Locating Codes in ICD-10 PCS Book...........................................................Error! Bookmark not defined.
ICD-10 PCS Characters: Definitions and Guidelines..................................Error! Bookmark not defined.
Definitions Used in ICD-10-PCS...................................................................Error! Bookmark not defined.
Character 1: Section ........................................................................................Error! Bookmark not defined.
Character 2: Body System .............................................................................Error! Bookmark not defined.
Character 3: Root Operation ........................................................................Error! Bookmark not defined.
Additional Root Operations ...............................................................Error! Bookmark not defined.
Root Operation Guidelines.................................................................Error! Bookmark not defined.
Character 4: Body Part ...................................................................................Error! Bookmark not defined.
Character 5: Approach ...................................................................................Error! Bookmark not defined.
Character 6: Device .........................................................................................Error! Bookmark not defined.
Character 7: Qualifier .....................................................................................Error! Bookmark not defined.
Scenarios: ICD-10-PCS Medical Surgical Section .......................................Error! Bookmark not defined.
General Overview of the Medical and Surgical-related Sections ..........Error! Bookmark not defined.
Measurement and Monitoring, Extracorporeal Assistance and Performance, and Extracorporeal
Therapies Sections ..........................................................................................Error! Bookmark not defined.
Osteopathic, Other Procedures, and Chiropractic Sections................Error! Bookmark not defined.

-Proprietary and Confidential-


©2014 himagine solutions inc.| 1001 East Palm Avenue | Tampa, FL 33605 | 866-230-7865 | ICD10@himaginesolutions.com
General Overview of the Ancillary Sections ............................................Error! Bookmark not defined.
Physical Rehabilitation and Diagnostic Audiology, Mental Health, and Substance Abuse Treatment
Sections...............................................................................................................Error! Bookmark not defined.
Scenarios Procedures in Medical and Surgical Related Sections ..........Error! Bookmark not defined.
Case Studies .................................................................................................................................................................... 87
Respiratory Disease ................................................................................................................................................. 87
Circulatory Disease .................................................................................................................................................. 88
Endocrine Disease .................................................................................................................................................... 91
Blood and Blood Forming ...................................................................................................................................... 92
Urinary/Kidney Disease ......................................................................................................................................... 94
Nervous System......................................................................................................................................................... 96
OB/GYN........................................................................................................................................................................ 97
Infection Disease ....................................................................................................................................................... 98
Scenario Answers: ICD-10-CM................................................................................................................................170
Scenario Answers: ICD-10-PCS........................................................................Error! Bookmark not defined.
Case Study Answers: ICD-10 CM & ICD-10-PCS ................................................................................................177
Resources and References........................................................................................................................................180

-Proprietary and Confidential-


©2014 himagine solutions inc.| 1001 East Palm Avenue | Tampa, FL 33605 | 866-230-7865 | ICD10@himaginesolutions.com
Introduction to ICD-10

-Proprietary and Confidential-


©2014 himagine solutions inc. P a g e |5
Introduction to ICD-10
The Department of Health and Human Services (HHS) final ruling that was announced and confirmed in
the Federal Register on August 15, 2008, states that all health care entities using ICD -9 diagnosis and
procedure codes must migrate to ICD-10 by October 1, 2015. The Final Rule revised January 2009, set
the planning and implementation parameters in a more finite manner. It has been estimated that
conversion to ICD-10 will require 3-5 times the effort and money than any other prior healthcare
conversion!

Even though there is still time for excitement and planning as with any new change there is a certain
amount of anxiety that is expected with such a major overhaul such as the ICD-10 system will bring with
it. So, the best way to alleviate the “fear of change or the fear of the unknown” is to meet it head on.
The best way to do this is by learning and preparing earlier than later for what is to come.

In this workbook you will find information in regards to ICD-10-CM and ICD-10-PCS. You will find great
detail in regards to the ICD-10 PCS characters including definitions, guidelines and examples.

Why is ICD-10 important to our ever evolving healthcare system at this juncture? It is important to note
the World Health Organization developed and released the ICD-10 coding system a decade ago and it is
currently used worldwide for classification of diseases and signs, symptoms, abnormal findings,
complaints, social circumstances and external causes of injury or diseases. The United States is the last
industrialized country in the world to implement ICD-10. The International Classification of Diseases
(ICD) 10th revision, allows enhancement for the efficiency of clinical data collection and the quality of
administrative data. ICD-9-CM is outdated without room for expansion to meet the needs of new
disease processes and utilizes antiquated terminology. The increased specificity related to the
classification of diseases allows for improved quality measures, research capability, value based
purchasing initiatives and cost efficiency.
There has been much controversy and anxiety related to the transition to ICD-10 due to costs required
to update systems and the exchange of healthcare information necessary for payment. In addition, the
necessary education for all involved in the coding and classification of disease processes is also costly
and time consuming. In many cases, anatomy and physiology, medical terminology, pathophysiology,
and pharmacology will have to be learned or require refreshing knowledge bases for these subjects
because coder knowledge of these topics haven’t been updated since their primary education. Needless
to say, it will be a “team effort” between providers, physicians and payers to make this transition as
smooth as possible.
One of the key areas we can influence during the next year is physician documentation. Understanding
the key differences in the classification system and how it impacts physician documentation of diseases
will help you communicate with the physician and prepare him/her for the increased specificity required
for ICD-10-CM. These sessions along with this workbook will be helpful in education of providers and
other service providers with documentation privileges in the patient record.

-Proprietary and Confidential-


©2014 himagine solutions inc. P a g e |6
Top Reasons We Need ICD-10 Now
 It Enhances Quality Measures. Without ICD-10 data, serious gaps will remain in the healthcare
community’s ability to extract important patient health information needed for physicians and
others to measure quality care.
 Research Capabilities Will Improve Patient Care. Data could be used in a more meaningful way
to enable better understanding of complications, better design of clinically robust algorithms,
and better tracking of the outcomes of care. Greater detail offers the ability to discover
previously-unrecognized relationships or uncover phenomenon such as incipient epidemics
early.
 Significant Progress Has Already Been Made. For several years, hospitals and healthcare
systems, health plans, vendors and academic institutions have been preparing in good faith to
put systems in place to transition to ICD-10. A delay would cause an unnecessary setback.
 Education Programs Are Underway. To ready the next generation of HIM professionals,
academic institutions have set their curriculum for two-year, four-year, and graduate programs
to include ICD-10.
 Other Healthcare Initiatives Need ICD-10. ICD-10 is the foundation needed to support other
national healthcare initiatives such as meaningful use, value-based purchasing, payment reform,
quality reporting and accountable care organizations. Electronic health record systems being
adopted today are ICD-10 compatible. Without ICD-10, the value of these other efforts is greatly
diminished.
 It Reduces Fraud. With ICD-10, the detail of health procedures will be easier to track, reducing
opportunities for unscrupulous practitioners to cheat the system.
 It Promotes Cost Effectiveness. More accurate information will reduce waste, lead to more
accurate reimbursement and help ensure that healthcare dollars are used efficiently.

Source; AHIMA Special E-ALERT February 22, 2012, CEO Lynne Thomas Gordon

-Proprietary and Confidential-


©2014 himagine solutions inc. P a g e |7
ICD-10-CM System

-Proprietary and Confidential-


©2014 himagine solutions inc. P a g e |8
ICD-10-CM System
The compliance date for implementation of the International Classification of Diseases, 10th Edition,
Procedure Coding System/Clinical Modification (ICD-10-PCS/CM) is currently October 1, 2015 for all
covered entities. ICD-10-CM, including the ICD-10-CM Official Guidelines for Coding and Reporting, will
replace the International Classification of Diseases, 9th Edition, Clinical Modification (ICD -9-CM)
diagnosis code set in all health care settings for diagnosis reporting with dates of service, or dates of
discharge for inpatients, currently occurring on or after October 1, 2015. This publication discusses the
benefits of ICD-10-CM, similarities and differences between the two coding systems, and new features
and additional changes that can be found in ICD-10-CM.

Benefits of ICD-10-CM
ICD-10-CM incorporates much greater clinical detail and specificity than ICD-9-CM. Terminology and
disease classification have been updated to be consistent with current clinical practice. The modern
classification system will provide much better data needed for:

 Measuring the quality, safety, and efficacy of care;


 Reducing the need for attachments to explain the patient’s condition;
 Designing payment systems and processing claims for reimbursement;
 Conducting research, epidemiological studies, and clinical trials;
 Setting health policy;
 Operational and strategic planning;
 Designing health care delivery systems;
 Monitoring resource utilization;
 Improving clinical, financial, and administrative performance;
 Preventing and detecting health care fraud and abuse; and
 Tracking public health and risks.

Non-specific codes still exist for use when the medical record documentation does not support a more
specific code.

Similarities and Differences between the two Coding Systems


ICD-10-CM uses 3–7 alpha and numeric characters and full code titles, but the format is very much the
same as ICD-9-CM (e.g., ICD-10-CM has the same hierarchical structure as ICD-9-CM).

The 7th character in ICD-10-CM is used in several chapters (e.g., the Obstetrics, Injury, Musculoskeletal,
and External Cause chapters). It has a different meaning depending on the section where it is being used
(e.g., in the Injury and External Cause sections, the 7th character classifies an initial encounter,
subsequent encounter, or sequelae (late effect)). Primarily, changes in ICD-10-CM are in its organization
and structure, code composition, and level of detail.

-Proprietary and Confidential-


©2014 himagine solutions inc. P a g e |9
ICD-9-CM Diagnoses Codes:
 3–5 digits;
 First digit is alpha (E or V) or numeric (alpha characters are not case sensitive);
 Digits 2–5 are numeric; and
 Decimal is used after third character.

ICD-10-CM Diagnosis Codes:


 3–7 characters;
 Character 1 is alpha; Character 2 is numeric;
 Characters 3–7 are alpha or numeric (alpha characters are not case sensitive); and
 Decimal is used after third character.

Structural Changes in ICD-10-CM:


ICD-10 will have the same hierarchical structure as ICD-9-CM but will have the following differences:

 ICD-10-CM has 21 chapters compared to 17 chapters in ICD-9-CM


 ICD-9-CM’s V and E code classifications are incorporated into the main classifications in ICD -10-
CM
 Diseases and conditions of the sense organs (eyes and ears) have been separated from the
nervous system diseases and conditions and have their own chapters in ICD-10-CM
 To reflect current knowledge, certain diseases have been reclassified (or reassigned) to a more
appropriate chapter in ICD-10-CM. For example, gout has been reclassified from the endocrine
chapter in ICD-9-CM to the musculoskeletal chapter in ICD-10-CM
 In contrast to ICD-9-CM, which classifies injuries by type, ICD-10-CM groups injuries first by
specific site (head, arm, leg, and such) and then by type of injury (fracture, open wound, etc.)
 Postoperative complications have been moved to procedure specific body system chapters
 ICD-10-CM codes are alphanumeric and can be up to seven characters in length
 ICD-10-CM includes full code titles for all codes (no reference back to common fourth and fifth
digits)
 Addition of sixth character in some chapters
 Addition of code extensions (seventh character) for obstetrics, injuries and external causes of
injuries
 Addition of dummy placeholder (X)

-Proprietary and Confidential-


©2014 himagine solutions inc. P a g e | 10
New features found in ICD-10-CM
The following new features can be found in ICD-10-CM:

 Laterality
 Combination Codes (for conditions with common manifestations, e.g., Crohn’s disease with
rectal bleeding, obstruction, etc.)
 Excludes Notes

 Excludes 1
 Excludes 2
 Expanded Codes (e.g., Non-pressure ulcer codes now delineate depth of ulcer (e.g., with fat layer
exposed, with necrosis of muscle, etc.)
 Injuries are grouped by anatomical site rather than by type of injury
 Category restructuring and code reorganization have occurred in a number of ICD-10-CM chapters,
resulting in the classification of certain diseases and disorders that are different from ICD-9-CM
 Certain diseases have been reclassified to different chapters or sections in order to reflect current
medical knowledge
 New code definitions (e.g., definition of acute myocardial infarction is now 4 weeks rather than 8
weeks)
 The codes corresponding to ICD-9-CM V codes (Factors Influencing Health Status and Contact with
Health Services) and E codes (External Causes of Injury and Poisoning) are incorporated into the main
classification rather than separated into supplementary classifications as they were in ICD-9-CM
 Placeholder character; The ICD-10-CM utilizes a placeholder character “X”. The “X” is used as a
placeholder at certain codes to allow for future expansion. An example of this is at the poisoning,
adverse effect and underdosing codes, categories T36-T50. Where a placeholder exists, the X must be
used in order for the code to be considered a valid code.
 7th Characters; Certain ICD-10-CM categories have applicable 7th characters. The applicable 7th
character is required for all codes within the category, or as the notes in the Tabular List instruct. The
7th character must always be the 7th character in the data field. If a code that requires a 7th
character is not 6 characters, a placeholder X must be used to fill in the empty characters.
 Episode of care for most injuries is incorporated into the injury codes

-Proprietary and Confidential-


©2014 himagine solutions inc. P a g e | 11
ICD-10-CM Guidelines

-Proprietary and Confidential-


©2014 himagine solutions inc. P a g e | 27
ICD-10-CM Guidelines
In 2011 the National Center for Health Statistics (NCHS) revised the draft ICD-10-CM Official Guidelines
for Coding and Reporting. These guidelines have been approved by the four organizations referred to as
the Cooperating Parties for ICD-10-CM: the American Hospital Association (AHA), the American Health
Information Management Association (AHIMA), the Centers for Medicare & Medicaid Services (CMS)
and NCHS. The guidelines are organized into four sections similar to the ICD-9-CM Official Guidelines for
Coding and Reporting. Section I includes the structure and conventions of the classification and general
guidelines that apply to the entire classification in addition to chapter-specific guidelines that
correspond to the chapters as they are arranged in the classification. Section II includes guidelines for
selection of principal diagnosis for non-outpatient settings. Section III includes guidelines for reporting
additional diagnoses in non-outpatient settings. Section IV is for outpatient coding and reporting.
The General Coding Guidelines (Part B of Section I) for ICD-10-CM are similar to their ICD-9-CM General
Coding Guidelines counterparts with the exception of few changes. All coding guidelines can be located
in the text of the Draft ICD-10 CM Code Book. Listed below are some ICD-10 CM guidelines that are
unique from what we have seen in ICD-9 CM however please review all ICD-10 guidelines in their
entirety. References for guidelines can be found in the reference section of this document.

ICD-10-CM Coding Guideline changes and additions


 Level of Detail in Coding

 Diagnosis codes are to be used and reported at their highest number of characters available. ICD-10-
CM diagnosis codes are composed of codes with 3, 4, 5, 6 or 7 characters. Codes with three
characters are included in ICD-10-CM as the heading of a category of codes that may be further
subdivided by the use of fourth and/or fifth characters and/or sixth characters, which provide greater
detail. A three-character code is to be used only if it is not further subdivided. A code is invalid if it
has not been coded to the full number of characters required for that code, including the 7th
character, if applicable.

 Laterality

 The Laterality guideline states “For bilateral sites, the final character of the codes in the ICD-10-CM
indicates laterality. An unspecified side code is also provided should the side not be identified in the
medical record. If no bilateral code is provided and the condition is bilateral, assign separate codes
for both the left and right side

-Proprietary and Confidential-


©2014 himagine solutions inc. P a g e | 28
 Punctuation

 [ ] Brackets are used in the Tabular List to enclose synonyms, alternative wording or explanatory
phrases. Brackets are used in the Alphabetic Index to identify manifestation codes. ( ) Parentheses
are used in both the Alphabetic Index and Tabular List to enclose supplementary words that may be
present or absent in the statement of a disease or procedure without affecting the code number to
which it is assigned. The terms within the parentheses are referred to as nonessential modifiers. The
nonessential modifiers in the Alphabetic Index to Disease apply to subterms following a main term
except when a nonessential modifier and a subentry are mutually exclusive, the subentry takes
precedence. For example, in the ICD-10-CM Alphabetic Index under the main term Enteritis, “acute”
is a nonessential modifier and “chronic” is a subentry. In this case, the nonessential modifier “acute”
does not apply to the subentry “chronic”.

 ICD-10-CM has added two excludes notes (Excludes 1 and Excludes 2)

 Excludes 1 note is a pure excludes note. It means “NOT CODED HERE!” An Excludes 1 note indicates
the code excluded should never be used when two conditions cannot occur together, such as a
congenital form versus an acquired form of the same condition.
 Excludes 2 note means “Not included here.” This type of excludes note indicates that the condition
excluded is not part of the condition represented by the code, but a patient may have both
conditions at the same time. When an Excludes 2 note appears under a code, it is acceptable to use
both the code and the excluded code together if the patient has both conditions.

 Chapter 2.c.1 and 2.c.2 (Anemia and malignancy) significant change in ICD-10-CM

 Guideline – 2.c.1 states Anemia associated with malignancy – When admission/encounter is for
management of an anemia associated with the malignancy, and the treatment is only for anemia, the
appropriate code for the malignancy is sequenced as the principal or first-listed diagnosis followed by
the appropriate code for the anemia (such as code D63.0, Anemia in neoplastic disease).
 Guideline – 2.c.2 states Anemia associated with chemotherapy, immunotherapy and radiation
therapy – When the admission/encounter is for management of an anemia associated with an
adverse effect of the administration of chemotherapy or immunotherapy and the only treatment is
for the anemia, the anemia code is sequenced first followed by the appropriate codes for the
neoplasm and the adverse effect (T45.1X5, Adverse effect of antineoplastic and immunosuppressive
drugs).

 Use of Sign/Symptom/Unspecified Codes

 Sign/symptom and “unspecified” codes have acceptable, even necessary, uses. While specific
diagnosis codes should be reported when they are supported by the available medical record
documentation and clinical knowledge of the patient’s health condition, there are instances when
signs/symptoms or unspecified codes are the best choices for accurately reflecting the healthcare

-Proprietary and Confidential-


©2014 himagine solutions inc. P a g e | 29
encounter. Each healthcare encounter should be coded to the level of certainty known for that
encounter.
 If a definitive diagnosis has not been established by the end of the encounter, it is appropriate to
report codes for sign(s) and/or symptom(s) in lieu of a definitive diagnosis. When sufficient clinical
information isn’t known or available about a particular health condition to assign a more specific
code, it is acceptable to report the appropriate “unspecified” code (e.g., a diagnosis of pneumonia
has been determined, but not the specific type). Unspecified codes should be reported when they are
the codes that most accurately reflects what is known about the patient’s condition at the time of
that particular encounter. It would be inappropriate to select a specific code that is not supported by
the medical record documentation or conduct medically unnecessary diagnostic testing in order to
determine a more specific code.

 Poisoning

 When coding a poisoning or reaction to the improper use of a medication (e.g., overdose, wrong
substance given or taken in error, wrong route of administration), first assign the appropriate code
from categories T36-T50. The poisoning codes have an associated intent as their 5th or 6th character
accidental, intentional self-harm, assault and undetermined. Use additional code(s) for all
manifestations of poisonings.

 Underdosing

 Underdosing refers to taking less of a medication than is prescribed by a provider or a manufacturer’s


instruction. For underdosing, assign the code from categories T36-T50 (fifth or sixth character “6”).
Codes for underdosing should never be assigned as principal or first-listed diagnosis. If a patient has a
relapse or exacerbation of the medical condition for which the drug is prescribed because of the
reduction in dose, then the medical condition itself should be coded. Noncompliance (Z91.12 -,
Z91.13-) or complications of care (Y63.61, Y63.8-Y63.9) codes are to be used with an underdosing
code to indicate intent, if known.

Additional chapter specific guidelines will be discussed throughout specific chapters


within the workbook.

-Proprietary and Confidential-


©2014 himagine solutions inc. P a g e | 30
ICD-10 Chapters 1-7

-Proprietary and Confidential-


©2014 himagine solutions inc. P a g e | 31
Organization and Classification of Diseases and
Disorders – Chapters 1–7
This section is designed to present an overview of the changes to specific disorders and conditions
classified to Chapters 1–7 of ICD-10-CM. Not every revision has been identified but certain conditions
have been selected to point out concepts that represent organizational, terminology, and classification
modifications.

Chapter 1: Certain infectious and parasitic diseases (A00-B99)


Chapter 1 of ICD-10-CM includes categories A00-B99 arranged in the following blocks:

 A00-A09 Intestinal infectious diseases


 A15-A19 Tuberculosis
 A20-A28 Certain zoonotic bacterial diseases
 A30-A49 Other bacterial diseases
 A50-A64 Infections with a predominantly sexual mode of transmission
 A65-A69 Other spirochetal diseases
 A70-A74 Other diseases caused by Chlamydiae
 A75-A79 Rickettsioses
 A80-A89 Viral and prion infections of the central nervous system
 A90-A99 Arthropod-borne viral fevers and viral hemorrhagic fevers
 B00-B09 Viral infections characterized by skin and mucous membrane lesions
 B10 Other human herpes viruses
 B15-B19 Viral hepatitis
 B20 Human immunodeficiency virus [HIV] disease
 B25-B34 Other viral diseases
 B35-B49 Mycoses
 B50-B64 Protozoal diseases
 B65-B83 Helminthiases
 B85-B89 Pediculosis, acariasis and other infestations
 B90-B94 Sequelae of infectious and parasitic diseases
 B95-B97 Bacterial, viral and other infectious agents
 B99 Other infectious diseases

-Proprietary and Confidential-


©2014 himagine solutions inc. P a g e | 32
While overall Chapter 1 of ICD-10-CM is organized similarly to ICD-9-CM, some category and
subcategory titles have been changed.

Certain diseases have also been rearranged in Chapter 1 of ICD-10-CM. For example, a separate
subchapter, or block, has been created and appropriate conditions grouped together for Infections with
a predominantly sexual mode of transmission (A50-A64). Two additional examples of separate blocks
being created with the appropriate conditions grouped together are viral hepatitis (B15-B19) and other
viral diseases (B25-B34).

Some terminology changes and revisions to the classification of specific infectious and parasitic disease
in ICD-10-CM have occurred as well. For instance, the term sepsis has replaced septicemia throughout
Chapter 1. Additionally, streptococcal sore throat and its inclusion terms found in the Infectious and
Parasitic Disease chapter of ICD-9-CM are reclassified in ICD-10-CM to Chapter 10: Diseases of the
respiratory system.

Many of the codes in Chapter 1 of ICD-10-CM have been expanded to reflect manifestations of the
disease with the use of fourth or fifth characters allowing the infectious disease and manifestation to be
captured in one code instead of two.

Infectious Disease Guidelines


At the beginning of Chapter 1 is an instruction to use an additional code for any associated drug
resistance (Z16). This new guideline should be followed when assigning any code from this chapter.
Guidelines for code usage may also be category specific.
The NCHS has published chapter-specific guidelines for Chapter 1 of ICD-10-CM:

 Guideline I.C.1.a. Human Immunodeficiency Virus [HIV] Infections


 Guideline I.C.1.b. Infectious agents as the cause of diseases classified to other chapters
 Guideline I.C.1.c .Infections resistant to antibiotics
 Guideline I.C.1.d. Sepsis, Severe Sepsis, and Septic Shock

Infectious Disease Scenarios


Scenario 1 (Coding):
Chronic vulvitis due to monilia with microorganisms resistant to cephalosporin.

____________________________________________________________________________________________

-Proprietary and Confidential-


©2014 himagine solutions inc. P a g e | 33
Scenario 2 (Coding):
Patient’s final diagnosis is gram negative sepsis with acute respiratory failure.

____________________________________________________________________________________________

Scenario 3 (Coding):
This patient is a 36-year-old HIV positive male who comes in with fever and cough. Final diagnosis was
Pneumocystis pneumonia due to AIDS.

____________________________________________________________________________________________

Chapter 2: Neoplasms (C00-D49)


Chapter 2 of ICD-10-CM includes categories C00-D49 arranged in the following blocks:

 C00-C75 Malignant neoplasms stated or presumed to be primary (of specific sites) and certain
specified histologies, except neuroendocrine, and of lymphoid, hematopoietic and related
tissues
 C00-C14 Malignant neoplasms of lip, oral cavity and pharynx
 C15-C26 Malignant neoplasms of digestive organs
 C30-C39 Malignant neoplasms of respiratory and intrathoracic organs
 C40-C41 Malignant neoplasms of bone and articular cartilage
 C43-C44 Melanoma and other malignant neoplasms of skin
 C45-C49 Malignant neoplasms mesothelial and soft tissue
 C50 Malignant neoplasms of breast
 C51-C58 Malignant neoplasms of female genital organs
 C60-C63 Malignant neoplasms of male genital organs
 C64-C68 Malignant neoplasms of urinary tract
 C69-C72 Malignant neoplasms of eye, brain and other parts of central nervous system
 C73-C75 Malignant neoplasms of thyroid and other endocrine glands
 C7A Malignant neuroendocrine tumors
 C7B Secondary neuroendocrine tumors
 C76-C80 Malignant neoplasms of ill-defined, other secondary and unspecified sites
 C81-C96 Malignant neoplasm of lymphoid, hematopoietic and related tissue
 D00-D09 In situ neoplasms
 D10-D36 Benign neoplasms except benign neuroendocrine tumors

-Proprietary and Confidential-


©2014 himagine solutions inc. P a g e | 34
 D3A Benign neuroendocrine tumors
 D37-D48 Neoplasms of uncertain behavior, polycythemia vera and myelodysplastic syndromes
 D49 Neoplasms of unspecified behavior
The neoplasm chapter has undergone some organizational changes, too. For example, in ICD-10-CM, the
block of codes for in situ neoplasms is located before the block for benign neoplasms. An example of a
classification improvement is the addition in ICD-10-CM of a separate fifth character for extranodal and
solid organ sites for lymphomas and Hodgkin’s. ICD-9-CM included these sites with the fifth digit for
unspecified site in codes for Hodgkin’s disease, non-Hodgkin’s lymphoma, peripheral, and cutaneous T-
cell lymphomas.

Neoplasm Guidelines
A new guideline under category D3A, Benign neuroendocrine tumor instructs the coding professional to
code additional disorders.

Another new guideline is found under many of the categories for malignant neoplasms. This guideline
instructs coding professionals to use an additional code to identify such conditions as alcohol abuse and
dependence, alcohol dependence in remission, tobacco dependence, and history of tobacco use.

The NCHS has published chapter-specific guidelines for Chapter 2 of ICD-10-CM. Some of these
guidelines are:

 Guidelines I.C.2. General neoplasm guidelines


 Guideline I.C.2.a. Treatment directed at the malignancy
 Guideline I.C.2.b. Treatment of secondary site
 Guidelines I.C.2.c. Coding and sequencing of complications
 Guideline I.C.2.d. Primary malignancy previously excised
 Guidelines I.C.2.e. Admissions/encounters involving chemotherapy, immunotherapy and
radiation therapy
 Guidelines I.C.2.l. Sequencing of neoplasm codes

Neoplasm Scenarios
Scenario 4 (Coding):
56-year-old male is being seen today for chemotherapy treatment. Patient has a diagnosis of carcinoma
of the small intestines, in the area where the duodenum and jejunum join. This carcinoma was totally
removed three months ago.

_________________________________________________________________________________ ___________

-Proprietary and Confidential-


©2014 himagine solutions inc. P a g e | 35
Scenario 5 (Coding):
Patient is admitted with malignant neoplasm of upper outer quadrant of left female breast with
metastasis to lymph axillary nodes.

____________________________________________________________________________________________

Scenario 6 (Coding):
A patient received scheduled chemotherapy treatment for lung cancer today in the hospital outpatient
department. The patient underwent removal of malignant nodule in the upper lobe of the right lung 4
weeks ago.

____________________________________________________________________________________________

Chapter 3: Diseases of the blood and blood-forming organs and certain


disorders involving the immune mechanism (D50-D89)
Chapter 3 of ICD-10-CM includes categories D50-D89 arranged in the following blocks:

 D50-D53 Nutritional anemias


 D55-D59 Hemolytic anemias
 D60-D64 Aplastic and other anemias and other bone marrow failure syndromes
 D65-D69 Coagulation defects, purpura and other hemorrhagic conditions
 D70-D77 Other disorders of blood and blood-forming organs
 D78 Intraoperative and postprocedural complications of spleen
 D80-D89 Certain disorders involving the immune mechanism

Coding professionals will find the organizational structure of ICD-10-CM’s Chapter 3 an improvement
over ICD-9-CM’s Chapter 4: Diseases of the Blood and Blood-forming Organs. Diseases and disorders
have been grouped into subchapters or blocks making it easier to identify the type of conditions
classified to Chapter 3. Modifications have also been made to specific categories that bring the
terminology up-to-date with current medical practice. Other enhancements to Chapter 3 include
classification changes that provide greater specificity than found in ICD-9-CM.

The last block in this chapter (D80-D89) groups disorders involving the immune mechanism. The
immunodeficiency disorders have been reclassified from Chapter 4: Endocrine, Nutritional and
Metabolic Diseases, and Immunity Disorders in ICD-9-CM to Chapter 3 in ICD-10-CM.

-Proprietary and Confidential-


©2014 himagine solutions inc. P a g e | 36
Blood and Blood Forming Organs and Certain Disorders Involving the Immune
Mechanism Scenarios Guidelines
At this time, there are no chapter-specific guidelines related to Chapter 3: Diseases of the blood and
blood-forming organs and certain disorders involving the immune mechanism.

Blood and Blood Forming Organs and Certain Disorders Involving the Immune
Mechanism Scenarios
Scenario 7 (Coding):
This 48-year-old female is seen for sickle-cell crisis with acute chest syndrome

____________________________________________________________________________________________

Scenario 8 (Coding):
This is an elderly woman being seen in the ER with complaints of nausea and not “feeling like eating.”
During the gastrointestinal work-up, mild gastritis was revealed. Lab work also indicated anemia,
probably due to the lack of nutrition. Admitted to inpatient stay.

____________________________________________________________________________________________

Scenario 9 (Coding):

This 25-year-old male was admitted to the hospital with pain symptoms related to sickle cell/HB-SS
disease. Chest x-ray was performed and revealed evidence of acute chest syndrome.

____________________________________________________________________________________________

Chapter 4: Endocrine, nutritional and metabolic diseases (E00-E89)


Chapter 4 of ICD-10-CM includes categories E00-E90 arranged in the following blocks:

 E00-E07 Disorders of thyroid gland


 E08-E13 Diabetes mellitus
 E15-E16 Other disorders of glucose regulation and pancreatic internal secretion
 E20-E35 Disorders of other endocrine glands
 E36 Intraoperative complications of endocrine system
 E40-E46 Malnutrition
 E50-E64 Other nutritional deficiencies

-Proprietary and Confidential-


©2014 himagine solutions inc. P a g e | 37
 E65-E68 Overweight, obesity and other hyperalimentation
 E70-E88 Metabolic disorders
 E89 Postprocedural endocrine and metabolic complications and disorders, not elsewhere
classified

A number of new subchapters have been added to the chapter for endocrine, nutritional, and metabolic
diseases. For example, diabetes mellitus and malnutrition have their own subchapter while these
conditions were grouped with diseases of other endocrine glands and nutritional deficiencies
respectively. Code titles have been revised in a number of places in Chapter 4.
A significant change to ICD-10-CM is the classification of diabetes mellitus. Instead of a single category
(250) as in ICD-9-CM, there are five categories for diabetes mellitus in ICD-10-CM. Additionally, diabetes
mellitus codes have been expanded to reflect manifestations and complications of the disease by using
fourth or fifth characters rather than by using an additional code to identify the manifestation. ICD-10-
CM classifies inadequately controlled, out of control, and poorly controlled diabetes mellitus to diabetes
mellitus, by type with hyperglycemia.

Endocrine, Nutritional and Metabolic Disease Guidelines


Instructions for coding “late effects” or sequelae have been expanded in Chapter 4 of ICD-10-CM. For
example, Excludes1 notes have been added to some categories between E50-E63 to indicate that the
sequelae of the nutritional deficiency are assigned a code from category E64.
New guidelines that clarify code usage are also found under specific codes.
No such note appears under the ICD-9-CM code 259.2 for this same condition.
The NCHS has published chapter-specific guidelines for Chapter 4 of ICD-10-CM:

 Guidelines I.C.4.a. Diabetes Mellitus

Endocrine, Nutritional and Metabolic Disease Scenarios


Scenario 10 (Coding):
The patient is a 31-year-old male seen for diabetic chronic kidney disease, stage 3. The patient has type 2
diabetes and takes insulin on a daily basis.

____________________________________________________________________________________________

-Proprietary and Confidential-


©2014 himagine solutions inc. P a g e | 38
Scenario 11 (Coding):
This is a 43-year-old female with a history of Type 1 diabetes mellitus on insulin. Her diabetes is
complicated by polyneuropathy. Patient has been non-compliant with insulin, and was admitted with a
blood glucose over 600.

____________________________________________________________________________________________

Chapter 5: Mental and behavioral disorders (F01-F99)


As explained earlier, Chapter 5 contains more subchapters, categories, subcategories, and codes than
ICD-9-CM. Rather than grouping by psychotic, nonpsychotic disorders, or mental retardation as in ICD -9-
CM, ICD-10-CM organizes mental and behavioral disorders in the following blocks:

 F01-F09 Mental disorders due to known physiological conditions


 F10-F19 Mental and behavioral disorders due to psychoactive substance use
 F20-F29 Schizophrenia, schizotypal, delusional and other non-mood
 psychotic disorders
 F30-F39 Mood [affective] disorders
 F40-F48 Anxiety, dissociative, stress-related, somatoform and other nonpsychotic
 mental disorders
 F50-F59 Behavioral syndromes associated with physiological disturbances and
 physical factors
 F60-F69 Disorders of adult personality and behavior
 F70-F79 Intellectual Disabilities
 F80-F89 Pervasive and specific developmental disorders
 F90-F98 Behavioral and emotional disorders with onset usually occurring in
 childhood and adolescence
 F99 Unspecified mental disorder

Changes were necessary in many parts of Chapter 5 because of outdated terminology. For example,
given what has been discovered in the past 20 years about the effects of nicotine, ICD-10-CM contains a
separate category F17 for nicotine dependence with subcategories to identify the specific tobacco
product and nicotine-induced disorders. ICD-9-CM has a single code, 305.1, for tobacco use disorder or
tobacco dependence.

The identification of the stage of the substance use, namely continuous or episodic, is not a part of ICD-
10-CM. A single ICD-10-CM code identifies not only the substance but also the disorder the substance
use induced. There continues to be codes for substance dependence “in remission.”

-Proprietary and Confidential-


©2014 himagine solutions inc. P a g e | 39
Mental and Behavioral Disorders Guidelines
As a result of the changes to the chapter on mental and behavioral disorders, the guidelines also were
revised to provide instructions for correct code use. For instance, under the different categories for
mental disorders due to known physiological conditions (F01-F09) is a note to code first the underlying
physiological condition.
There are also guideline changes to a number of codes in this chapter.
By contrast to the note above, the ICD-9-CM code 316 has a note which instructs coding professionals to
use an additional code to identify the associated physical condition.
The NCHS has published chapter-specific guidelines for Chapter 5 of ICD-10-CM:

 Guideline I.C.5.a. Pain disorders related to psychological factors


 Guidelines I.C.5.b. Mental and behavioral disorders due to psychoactive substance use

Mental and Behavioral Disorders Scenarios


Scenario 12 (Coding):
This 18-year-old male has been drinking since he was 13. He was brought into the ER and then admitted
because of acute alcohol intoxication. Blood alcohol level shows 22mg/100 ml. The discharge diagnosis is
acute and chronic alcoholism, continuous. What diagnosis codes are assigned?

____________________________________________________________________________________________

Scenario 13 (Coding):
This is a 27-year-old female patient who was brought in to the emergency department by police after
becoming disorderly and aggressive at a restaurant. She admits to a history of paranoid schizophrenia.

Diagnoses: Schizophrenia, paranoid type, chronic with acute exacerbation. Borderline personality
disorder.

____________________________________________________________________________________________

Scenario 14 (Coding):
This is a 33-year-old male brought to the emergency room following a seizure. Family member states
that he is a heavy drinker. Final Diagnosis: Alcohol Withdrawal.

________________________________________________________________________ ____________________

-Proprietary and Confidential-


©2014 himagine solutions inc. P a g e | 40
Chapter 6: Diseases of the nervous system (G00-G99)
Chapter 6 of ICD-10-CM includes categories G00-G99 arranged in the following blocks:

 G00-G09 Inflammatory diseases of the central nervous system


 G10-G14 Systemic atrophies primarily affecting the central nervous system
 G20-G26 Extrapyramidal and movement disorders
 G30-G32 Other degenerative diseases of the nervous system
 G35-G37 Demyelinating diseases of the central nervous system
 G40-G47 Episodic and paroxysmal disorders
 G50-G59 Nerve, nerve root and plexus disorders
 G60-G65 Polyneuropathies and other disorders of the peripheral nervous system
 G70-G73 Diseases of myoneural junction and muscle
 G80-G83 Cerebral palsy and other paralytic syndromes
 G89-G99 Other disorders of the nervous system

The organization of Chapter 6 in ICD-10-CM is comparable to that in ICD-9-CM. One change to note is
that only diseases of the nervous system are contained in Chapter 6 of ICD-10-CM. Diseases of the sense
organs, namely eye/adnexa and ear/mastoid processes, each have their own chapter in ICD-10-CM while
they are combined into a single chapter in ICD-9-CM. A few categories in Chapter 6 have rephrased titles
and in some cases encompass a combination of conditions. Additionally, a number of codes for diseases
of the nervous system have been expanded in ICD-10-CM.

Diseases of the Nervous System Guidelines


No instructional note is found at the start of Chapter 6 in ICD-9-CM. However, this is not the case in ICD-
10-CM. A series of excluded conditions are listed that are applicable to all conditions classifiable to
Chapter 6.

Additional guideline modifications were made to specific codes.


By contrast to the above note, below ICD-9-CM category 338, Pain, not elsewhere classified, the note
instructs coding professionals to “use additional code to identify pain associated with psychological
factors (307.89).”
The NCHS has published chapter-specific guidelines for Chapter 6 of ICD-10-CM:

 Guideline I.C.6.a. Dominant/nondominant side


 Guidelines I.C.6.b. Pain – Category G89

-Proprietary and Confidential-


©2014 himagine solutions inc. P a g e | 41
Diseases of the Nervous System Scenarios
Scenario 15 (Coding):
This patient, a 15-year-old female, is being seen for management of juvenile myoclonic epilepsy. The
patient did not respond to treatment and was diagnosed with an intractable seizure.

____________________________________________________________________________________________

Scenario 16 (Coding):
Patient is seen today for hemiparesis of the right side. She has had this status for over a year.

____________________________________________________________________________________________

Scenario 17 (Coding):
This is a 72-year-old female patient who has been experiencing episodes of memory loss and loss of time
and place and sometimes does not recognize family. The patient has been experiencing these episodes
gradually over the past year, but in the past two months, her condition has worsened. In the past few
weeks, the patient has been getting aggressive and abusive with family. Patient was diagnosed with
Alzheimer’s Dementia.

____________________________________________________________________________________________

Chapter 7: Diseases of eyes and adnexa (H00-H59)


Chapter 7 of ICD-10-CM includes categories H00-H59 arranged in the following blocks:

 H00-H05 Disorders of eyelid, lacrimal system and orbit


 H10-H11 Disorders of conjunctiva
 H15-H22 Disorders of sclera, cornea, iris and ciliary body
 H25-H28 Disorders of lens
 H30-H36 Disorders of choroid and retina
 H40-H42 Glaucoma
 H43-H44 Disorders of vitreous body and globe
 H46-H47 Disorders of optic nerve and visual pathways
 H49-H52 Disorders of ocular muscles, binocular movement, accommodation and refraction
 H53-H54 Visual disturbances and blindness
 H55-H57 Other disorders of eye and adnexa

-Proprietary and Confidential-


©2014 himagine solutions inc. P a g e | 42
 H59 Intraoperative and postprocedural complications and disorders of eye and adnexa, not
elsewhere classified

Chapter 7 is an entirely new chapter in ICD-10-CM. In ICD-9-CM, the conditions classified in this chapter
are located in Chapter 6: Diseases of the Nervous System and Sense Organs. Chapter 7 in ICD-10-CM also
has a different organization than what is found in ICD-9-CM. While the structure is still by “site” for
diseases of the eye and adnexa, the order differs.

Some categories in Chapter 7 have undergone title changes to reflect the terminology used today. For
example, ICD-9-CM uses senile cataract while ICD-10-CM utilizes the descriptor age-related cataract.
Many of the classification changes in Chapter 7 have to do with the expansion of characters to provide
for laterality. ICD-10-CM contains codes for right side, left side, and in some instances bilateral sides for
diseases of the eye and adnexa.

Diseases of the Eye and Adnexa Guidelines


With the formation of a new chapter for Diseases of the eye and adnexa, come new instructions on
which conditions are excluded from Chapter 7 of ICD-10-CM. Since the placement of the note is at the
beginning of Chapter 7, none of the listed conditions would be coded here.
Included in this chapter are a number of guidelines for code usage that are code specific.
At this time, there are no chapter-specific guidelines related to Chapter 7: Diseases of the eye and
adnexa.

Diseases of the Eye and Adnexa Scenarios


Scenario 18 (Coding)
This 40-year-old woman presents to her physician with bilateral eye pain. Her condition is diagnosed as
non-ulcerative bilateral blepharitis of the upper eyelids. What is the correct diagnosis in this case?

____________________________________________________________________________________________

Scenario 19 (Coding):
This is a 56-year-old female patient with long-standing, primary open-angle glaucoma and age-related
bilateral macular degeneration. Patient is admitted because of abnormally high intraocular pressure.

Diagnosis: Acute Primary open-angle glaucoma macular degeneration

____________________________________________________________________________________________

-Proprietary and Confidential-


©2014 himagine solutions inc. P a g e | 43
ICD-10 Chapters 8-14

-Proprietary and Confidential-


©2014 himagine solutions inc. P a g e | 44
Organization and Classification of Diseases and
Disorders – Chapters 8–14
This section is designed to present an overview of the changes to specific disorders classified to Chapters
8–14 of ICD-10-CM. Not every revision has been identified but certain conditions have been selected to
point out concepts that represent organizational, terminology, and classification modifications.

Chapter 8: Diseases of the ear and mastoid process (H60-H95)


Chapter 8 of ICD-10-CM includes categories H60-H95 arranged in the following blocks:

 H60-H62 Diseases of external ear


 H65-H75 Diseases of middle ear and mastoid
 H80-H83 Diseases of inner ear
 H90-H94 Other disorders of ear
 H95 Intraoperative and postprocedural complications and disorders of ear and mastoid process,
not elsewhere classified

Chapter 8 is an entirely new chapter in ICD-10-CM. In ICD-9-CM, the conditions classified in this chapter
are located in Chapter 6: Diseases of the Nervous System and Sense Organs. Diseases of the ear and
mastoid process have been arranged into blocks making it easier to identify the types of conditions t hat
would occur in the external ear (block 1), middle ear and mastoid (block 2), and inner ear (block 3). Block
4 is used for other disorders of the ear. Block 5 contains the codes for intraoperative and postprocedural
complications. The intraoperative and postprocedural complications are grouped at the end of the
chapter rather than scattered throughout different categories. Category and subcategory titles have
been revised in a number of locations in Chapter 8.

Although Chapter 8 in ICD-10-CM basically parallels the corresponding section in Chapter 6 of ICD-9-CM,
there are quite a few changes. These changes include greater specificity added at the fourth-, fifth- and
sixth-character levels; the delineation of laterality and the addition of many more “code first underlying
disease” notes.
One last noted classification change in this chapter is that the ICD-9-CM category 381, Nonsuppurative
otitis media and Eustachian tube disorders, has been split into two categories in ICD -10-CM; H65,
Nonsuppurative otitis media and H68, Eustachian salpingitis and obstruction.

Diseases of the Ear and Mastoid Process Guidelines


Guidelines on the use of codes may change from one revision to the next. For example, ICD-9-CM
contains a note excluding otitis media with perforation of tympanic membrane from subcategory 384.2.

-Proprietary and Confidential-


©2014 himagine solutions inc. P a g e | 45
In ICD-10-CM the note directly under H72, Perforation of tympanic membrane, states “code first any
associated otitis media.”

Another new guideline is found under the category for suppurative and unspecified otitis media (H66).
The note instructs coding professionals to use an additional code to identify: exposure to environmental
tobacco smoke (Z77.22), exposure to tobacco smoke in the perinatal period (P96.81), history of tobacco
use (Z87.891), occupational exposure to environmental tobacco smoke (Z57.31), tobacco dependence
(F17.-), or tobacco use (Z72.0).

At this time, there are no chapter-specific guidelines related to Chapter 8: Diseases of the ears and
mastoid process.

Diseases of the Ear and Mastoid Process Scenarios


Scenario 20 (Coding):
Recurrent acute serous otitis media, right ear.

____________________________________________________________________________________________

Scenario 21(Coding):
The 42-year-old male has bilateral conductive hearing loss due to nonobliterative conductive hearing loss
due to nonobliterative otosclerosis of the staples at the oval window. He is unable to hear with hearing
aids and has decided to undergo left stapedectomy. During surgery an inadvertent laceration was made
to the tympanic meatal flap which was repaired.

____________________________________________________________________________________________

Chapter 9: Diseases of the circulatory system (I00-I99)


Chapter 9 of ICD-10-CM includes categories I00-I99 arranged in the following blocks:

 I00-I02 Acute rheumatic fever


 I05-I09 Chronic rheumatic heart diseases
 I10-I15 Hypertensive diseases
 I20-I25 Ischemic heart diseases
 I26-I28 Pulmonary heart disease and diseases of pulmonary circulation
 I30-I52 Other forms of heart disease
 I60-I69 Cerebrovascular diseases
 I70-I79 Diseases of arteries, arterioles and capillaries

-Proprietary and Confidential-


©2014 himagine solutions inc. P a g e | 46
 I80-I89 Diseases of veins, lymphatic vessels and lymph nodes, not elsewhere classified
 I95-I99 Other and unspecified disorders of the circulatory system

The organization of Chapter 9 in ICD-10-CM is comparable to Chapter 7 in ICD-9-CM. One change to note
is the order of conditions within the block for ischemic heart disease.

One last noted change is with the classification of hypertension. In ICD-9-CM, hypertension codes
classify the type of hypertension (benign, malignant, unspecified). In ICD-10-CM, hypertension codes no
longer classify the type.

Diseases of the Circulatory System Guidelines


There are guideline changes to a number of codes in this chapter. For example, in ICD-10-CM beneath
categories I21, I22, and I23 for acute myocardial infarction are notes that state how the specific category
must be used within four weeks (28 days) or less from onset. However, below category 410 in ICD-9-CM
the note refers to an 8 week or less time period.

New guidelines that clarify code usage are also found under specific codes. Under code I05, rheumatic
mitral valve diseases, is a note that states this category includes conditions classifiable to both I05.0 and
I05.2-I05.9, whether specified as rheumatic or not.
The NCHS has published chapter-specific guidelines for Chapter 9 of ICD-10-CM:

 Guidelines I.C.9.a. Hypertension


 Guideline I.C.9.b. Atherosclerotic coronary artery disease and angina
 Guideline I.C.9.c. Intraoperative and postprocedural cerebrovascular accident
 Guidelines I.C.9.d. Sequelae of cerebrovascular disease
 Guidelines I.C.9.e. Acute myocardial infarction

Diseases of the Circulatory System Scenarios


Scenario 22 (Coding):
Atherosclerosis of native coronary arteries with exertional angina.

____________________________________________________________________________________________

Scenario 23 (Coding):
Pulmonary Endarteritis

____________________________________________________________________________________________

-Proprietary and Confidential-


©2014 himagine solutions inc. P a g e | 47
Scenario 24 (Coding):
Acute MI, anterior wall, left anterior descending artery.

____________________________________________________________________________________________

Scenario 25 (Coding):
Acute Cerebrovascular infarction-embolism of the left cerebral artery with dysphagia and right
hemiplegia, dominant side. What diagnosis codes are assigned?

_________________________________________________________________ ___________________________

Scenario 26 (Coding):
Patient presented to the emergency department two weeks after being admitted for a previous MI and
was diagnosed with an acute inferior wall myocardial infarction. She is still being monitored following
her initial heart attack three weeks earlier and continues to have atrial fibrillation. She will be transferred
to a larger facility for a cardiac catheterization and possible further intervention. Please assign diagnosis
codes.

_______________________________________________________________________________ _____________

Scenario 27 (Coding):
This 54-year-old female is being treated for an acute non-ST anterior wall myocardial infarction which
she suffered 5 days ago. She also has atrial fibrillation.

____________________________________________________________________________________________

Chapter 10: Diseases of the respiratory system (J00-J99)


Chapter 10 of ICD-10-CM includes categories J00-J99 arranged in the following blocks:

 J00-J06 Acute upper respiratory infections


 J09-J18 Influenza and pneumonia
 J20-J22 Other acute lower respiratory infections
 J30-J39 Other diseases of upper respiratory tract
 J40-J47 Chronic lower respiratory diseases

-Proprietary and Confidential-


©2014 himagine solutions inc. P a g e | 48
 J60-J70 Lung diseases due to external agents
 J80-J84 Other respiratory diseases principally affecting the interstitium
 J85-J86 Suppurative and necrotic conditions of the lower respiratory tract
 J90-J94 Other diseases of the pleura
 J95 Intraoperative and postprocedural complications and disorders of respiratory system, not
elsewhere classified
 J96-J99 Other diseases of the respiratory system
While overall Chapter 10 of ICD-10-CM is organized similar to ICD-9-CM, diseases have been rearranged.
Modifications have also been made to specific categories that bring the terminology up-to-date with
current medical practice.
Other enhancements to Chapter 10 include classification changes that provide greater specificity than
found in ICD-9-CM.

Diseases of the Respiratory System Guidelines


At the beginning of Chapter 10 the following instructional guideline appears: “When a respiratory
condition is described as occurring in more than one site and is not specifically indexed, it should be
classified to the lower anatomic site.”
An additional instructional guideline also appears at the beginning of Chapter 10 which inst ructs the
coding professional to use an additional code, where applicable to identify: exposure to environmental
tobacco smoke (Z77.22), exposure to tobacco smoke in the perinatal period (P96.81), history of tobacco
use (Z87.891), occupational exposure to environmental tobacco smoke (Z57.31), tobacco dependence
(F17.-), or tobacco use (Z72.0).
Guidelines for code usage may also be category specific. Under ICD-10-CM category J10, influenza, is a
note to use an additional code to identify the virus. No such note appears under the ICD-9-CM category
(487) for this same condition.
The NCHS has published chapter-specific guidelines for Chapter 10 of ICD-10-CM:

 Guideline I.C.10.a. Chronic obstructive pulmonary disease and asthma


 Guidelines I.C.10.b. Acute respiratory failure
 Guideline I.C.10.c. Influenza due to certain identified influenza viruses
 Guidelines I.C.10.d. Ventilator-associated pneumonia

-Proprietary and Confidential-


©2014 himagine solutions inc. P a g e | 49
Diseases of the Respiratory System Scenarios
Scenario 28 (Coding):
Group A Streptococcal Tonsillitis

____________________________________________________________________________________________

Scenario 29 (Coding):
Chronic Obstructive Bronchitis

____________________________________________________________________________________________

Scenario 30 (Coding):
Acute Respiratory Failure

____________________________________________________________________________________________

Scenario 31 (Coding):
Discharge diagnosis: Moderate persistent asthma with status asthmaticus, Acute exacerbation of chronic
obstructive pulmonary disease. Hospital Course: The patient presented with gradual increase in
shortness of breath, which was unresponsive to home nebulizer treatments. In the emergency room, he
received more respiratory treatments; however, he failed to improve. Therefore, the patient was
admitted to the hospital. At the time of admission, the theophylline level was 5.9. Chest x-ray showed no
evidence of active infiltrates. The patient was bolused with intravenous steroids and started on frequent
respiratory therapy treatments. IV aminophylline boluses and drip were used to increase his theophylline
level to therapeutic range. The patient gradually cleared and by the next day was much better. His IV
aminophylline was changed to p.o. The Ventolin treatments were decreased to q 4 hr. and his steroids
were rapidly tapered back to 10 mg. of Prednisone. What diagnosis codes are assigned?

____________________________________________________________________________________________

Chapter 11: Diseases of the digestive system (K00-K94)


Chapter 11 of ICD-10-CM includes categories K00-K94 arranged in the following blocks:

 K00-K14 Diseases of oral cavity and salivary glands


 K20-K31 Diseases of esophagus, stomach and duodenum

-Proprietary and Confidential-


©2014 himagine solutions inc. P a g e | 50
 K35-K38 Diseases of appendix
 K40-K46 Hernia
 K50-K52 Noninfective enteritis and colitis
 K55-K64 Other diseases of intestines
 K65-K68 Diseases of peritoneum and retroperitoneum
 K70-K77 Diseases of liver
 K80-K87 Disorders of gallbladder, biliary tract and pancreas
 K90-K95 Other diseases of digestive system

A number of new subchapters have been added to the chapter for diseases of the digestive system. For
instance, in ICD-10-CM diseases of the liver have their own subchapter or block while these conditions
were grouped with other diseases of the digestive system in ICD-9-CM. Some terminology changes and
revisions to the classification of specific digestive conditions have occurred in ICD-10-CM as well.

ICD-10-CM category K50, Crohn’s disease, has been expanded to the fourth, fifth, and sixth character in
contrast to ICD-9-CM category 555, Regional enteritis. The expansion at the fourth character level
specifies the site of the Crohn’s disease, the fifth character indicates whether a complication was
present and the sixth character further classifies the specific complication.

In ICD-9-CM, the presence or absence of obstruction is used as an axis for classifying ulcers. In ICD -10-
CM a fairly substantial classification change was made and the identification of obstruction is no longer a
part of the ICD-10-CM ulcer code structure.

Diseases of the Digestive System Guidelines


Guideline modifications were made to specific codes in this chapter. For example, in ICD-9-CM ulcerative
colitis does not have any instructions for code usage listed below category 556. In contrast, guidelines
for category K51, Ulcerative colitis, state to use an additional code to identify manifestations.
No instructional note is found at the start of the subchapter for hernias in ICD-9-CM. However, this is
not the case in ICD-10-CM. The note “hernia with both gangrene and obstruction is classified to hernia
with gangrene” applies to all conditions coded to categories K40 through K46.

At this time, there are no chapter-specific guidelines related to Chapter 11: Diseases of the digestive
system.

-Proprietary and Confidential-


©2014 himagine solutions inc. P a g e | 51
Diseases of the Digestive System Scenarios
Scenario 32 (Coding):
Incarcerated Recurrent Inguinal Hernia, Right.

_______________________________________________________________ _____________________________

Scenario 33 (Coding):
Acute cholecystitis w/ stone in Gallbladder.

____________________________________________________________________________________________

Scenario 34 (Coding):
Acute suppurative peritonitis due to E Coli.

____________________________________________________________________________________________

Scenario 35 (Coding):
A 68-year-old man was admitted to the hospital for bilateral inguinal hernia repair that could not be
done on an outpatient basis because of anticipated extended recovery time required due to his COPD,
chronic low back pain, and hypertension. After being prepared for surgery, the patient complained of
precordial chest pain. The surgery was cancelled and the patient was returned to his room. Cardiac
studies failed to find a reason for the chest pain which resolved later that day. What are the correct
diagnosis codes?

____________________________________________________________________________________________

Chapter 12: Diseases of skin and subcutaneous tissue (L00-L99)


Chapter 12 of ICD-10-CM includes categories L00-L99 arranged in the following blocks:

 L00-L08 Infections of the skin and subcutaneous tissue


 L10-L14 Bullous disorders
 L20-L30 Dermatitis and eczema
 L40-L45 Papulosquamous disorders
 L49-L54 Urticaria and erythema
 L55-L59 Radiation-related disorders of the skin and subcutaneous tissue
 L60-L75 Disorders of skin appendages

-Proprietary and Confidential-


©2014 himagine solutions inc. P a g e | 52
 L76 Intraoperative and postprocedural complications of skin and subcutaneous tissue
 L80-L99 Other disorders of the skin and subcutaneous tissue

ICD-10-CM Chapter 12 represents a complete restructuring to bring together groups of diseases that are
related to one another in some way. Additionally, greater specificity has been added to many of the
codes at either the fourth-, fifth- and even sixth-character level. ICD-9-CM Chapter 12 has only three
subchapters which have been expanded in ICD-10-CM to create the nine blocks listed above.

One example of an organizational change to Chapter 12 of ICD-10-CM is a subchapter or block for ICD-
10-CM codes for radiation-related disorders of the skin and subcutaneous tissue. The conditions found
in this block are not located together in ICD-9-CM.

Some categories in Chapter 12 have undergone title changes to reflect terminology in use today. For
instance, ICD-10-CM uses androgenic alopecia while this term is not used at all in ICD-9-CM.
An example of a classification improvement is the addition of characters in ICD-10-CM to represent the
site and severity of the decubitus ulcer.

Diseases of the skin and subcutaneous tissue Guidelines


Instructions for coding dermatitis and eczema have been expanded in Chapter 12. For example, the note
“in this block the terms dermatitis and eczema are used synonymously and interchangeably” has been
added to categories L20-L30. Additionally, the “excludes” note has been expanded in ICD-10-CM for
categories L20-L30 compared to categories 690–698 in ICD-9-CM.

With the formation of new categories for allergic (L23) and irritant (L24) contact dermatitis, new
instructions on which condition to code first, that is, the drug or substance, have also been created.

This instruction is different from the one found under ICD-9-CM code 692.3, dermatitis due to drugs and
medicines in contact with the skin, which states to use an additional E code to identify the drug.
The NCHS has published chapter-specific guidelines for Chapter 12 of ICD-10-CM:

 Guidelines I.C.12.a. Pressure ulcer stage codes

Diseases of the skin and subcutaneous tissue scenarios


Scenario 36 (Coding):
Psoriatic arthropathy

____________________________________________________________________________________________

-Proprietary and Confidential-


©2014 himagine solutions inc. P a g e | 53
Scenario 37 (Coding):
Cellulitis, Right Thigh

____________________________________________________________________________________________

Scenario 38 (Coding):
Stage IV Decubitus Ulcer, Left Hip

____________________________________________________________________________________________

Scenario 39 (Coding):
This 35-year-old male patient presents with edema, redness, and pain of the left big toe. He did not seek
treatment because he thought it would improve on its own. He does not remember any injury, but the
pain has gotten progressively worse for the past week. Diagnosis: Gangrenous abscess of the entire left
big toe. What diagnosis codes are assigned?

____________________________________________________________________________________________

Chapter 13: Diseases of the musculoskeletal system and connective


tissue (M00-M99)
Chapter 13 contains many more subchapters, categories, and codes than ICD-9-CM. Rather than having
just four subchapters grouping many conditions together, ICD-10-CM organizes the musculoskeletal
system and connective tissue in the following blocks:

 M00-M02 Infectious arthropathies


 M05-M14 Inflammatory polyarthropathies
 M15-M19 Osteoarthritis
 M20-M25 Other joint disorders
 M26-M27 Dentofacial anomalies [including malocclusion] and other disorders of jaw
 M30-M36 Systemic connective tissue disorders
 M40-M43 Deforming dorsopathies
 M45-M49 Spondylopathies
 M50-M54 Other dorsopathies
 M60-M63 Disorders of muscles
 M65-M67 Disorders of synovium and tendon

-Proprietary and Confidential-


©2014 himagine solutions inc. P a g e | 54
 M70-M79 Other soft tissue disorders
 M80-M85 Disorders of bone density and structure
 M86-M90 Other osteopathies
 M91-M94 Chondropathies
 M95 Other disorders of the musculoskeletal system and connective tissue
 M96 Intraoperative and postprocedural complications and disorders of musculoskeletal system,
not elsewhere classified
 M99 Biomechanical lesions, not elsewhere classified

Almost every code in Chapter 13 of ICD-10-CM has been expanded in some way with the expansion
including very specific sites as well as laterality. Numerous codes have been moved from various
chapters in ICD-9-CM to Chapter 13 in ICD-10-CM.

Category M80 in ICD-10-CM classifies the type of osteoporosis in addition to the site of a current
pathological fracture into one combination code.
Additionally, some categories and subcategories in Chapter 13 require the use of seventh character
extensions.
The seventh character extensions are:

 A initial encounter for fracture


 B subsequent encounter for fracture with routine healing
 G subsequent encounter for fracture with delayed healing
 K subsequent encounter for fracture with nonunion
 P subsequent encounter for fracture with malunion
 S sequelae

Diseases of the Musculoskeletal System and Connective Tissue Guidelines


The first block of the chapter on diseases of the musculoskeletal system and connective tissue for
infectious arthropathies includes arthropathies due to microbiological agents. To assist coding
professionals on the correct usage of categories M00-M02, new guidelines provide definitions for direct
and indirect infection.

Instructional notes have also been added to different categories or subcategories to explain how codes
should be assigned.
Notes have also been used to define terms.
The NCHS has published chapter-specific guidelines for Chapter 13 of ICD-10-CM:

-Proprietary and Confidential-


©2014 himagine solutions inc. P a g e | 55
 Guideline I.C.13.a. Site and laterality
 Guideline I.C.13.b. Acute traumatic versus chronic or recurrent musculoskeletal conditions
 Guideline I.C.13.c. Coding of pathologic fractures
 Guidelines I.C.13.d. Osteoporosis

Diseases of the Musculoskeletal System and Connective Tissue Scenarios


Scenario 40 (Coding):
Recurrent Dislocation, Right Shoulder

____________________________________________________________________________________________

Scenario 41 (Coding):
Old Bucket Handle Tear, Lateral Meniscus, Right Knee

____________________________________________________________________________________________

Scenario 42 (Coding):
Nonunion Pathological Distal Right Humerus

____________________________________________________________________________________________

Scenario 43 (Coding):
This young man is being treated for his ongoing juvenile rheumatoid arthritis. This condition is found only
in both ankles and feet. What diagnosis codes are assigned?

____________________________________________________________________________________________

Chapter 14: Diseases of the genitourinary system (N00-N99)


Chapter 14 of ICD-10-CM includes categories N00-N99 arranged in the following blocks:

 N00-N08 Glomerular diseases


 N10-N16 Renal tubulo-interstitial diseases
 N17-N19 Acute kidney failure and chronic kidney disease
 N20-N23 Urolithiasis
 N25-N29 Other disorders of kidney and ureter

-Proprietary and Confidential-


©2014 himagine solutions inc. P a g e | 56
 N30-N39 Other diseases of the urinary system
 N40-N53 Diseases of male genital organs
 N60-N65 Disorders of breast
 N70-N77 Inflammatory diseases of female pelvic organs
 N80-N98 Noninflammatory disorders of female genital tract
 N99 Intraoperative and postprocedural complications and disorders of genitourinary
system, not elsewhere classified

For the most part, those genitourinary disorders in diseases classified elsewhere have been placed in
their own category at the end of each block of Chapter 14. This differs from ICD-9-CM in that these
conditions were classified within different subcategories.

Changes were necessary in some sections of Chapter 14 because of outdated terminology. For example,
given what has been discovered since the last revision of ICD about male erectile dysfunction, ICD-10-
CM includes category N52 for this condition with subcategories to identify the different causes of the
dysfunction. ICD-9-CM has a single code, 607.84, for impotence of organic origin.

Diseases of the Genitourinary System Guidelines


Throughout Chapter 14 are new includes notes that help to clarify the types of disorders that are
classified to the various categories.
A similar change has occurred to the instruction for menopausal and other perimenopausal disorders. In
ICD-9-CM, there is no guideline under category 627 to help coding professionals in their selection of a
code for these disorders. However, ICD-10-CM includes a note stating menopausal and other
perimenopausal disorders due to naturally occurring (age-related) menopause and perimenopause are
classified to category N95.
The NCHS has published chapter-specific guidelines for Chapter 14 of ICD-10-CM:

 Guidelines I.C.14.a. Chronic kidney disease

Diseases of the Genitourinary System Scenarios


Scenario 44 (Coding):
Pelvic Inflammatory Disease

____________________________________________________________________________________________

-Proprietary and Confidential-


©2014 himagine solutions inc. P a g e | 57
Scenario 45 (Coding):
Urinary Tract Infection due to E Coli

_____________________________________________________________________________ _______________

Scenario 46 (Coding):
Chronic Interstitial Cystitis with Hematuria

____________________________________________________________________________________________

Scenario 47 (Coding):
This 45-year-old female is currently being treated for chronic kidney disease, stage 3. She has previously
undergone a kidney transplant but still continues to suffer from chronic kidney disease. This patient is
also treated for hypothyroidism following removal of the thyroid for thyroid carcinoma. At this time,
there is no longer evidence of an existing thyroid malignancy. What diagnosis codes are assigned?

____________________________________________________________________________________________

-Proprietary and Confidential-


©2014 himagine solutions inc. P a g e | 58
ICD-10 Chapters 15-21

-Proprietary and Confidential-


©2014 himagine solutions inc. P a g e | 59
Organization and Classification of Diseases and
Disorders – Chapters 15–21
This section is designed to present an overview of the changes to specific disorders classified to Chapters
15–21 of ICD-10-CM. Not every revision has been identified but certain conditions have been selected to
point out concepts that represent organizational, terminology, and classification modifications.

Chapter 15: Pregnancy, childbirth and the puerperium (O00-O9A)


Chapter 15 of ICD-10-CM includes categories O00–O9A arranged in the following blocks:

 O00-O08 Pregnancy with abortive outcome


 O09 Supervision of high risk pregnancy
 O10-O16 Edema, proteinuria and hypertensive disorders in pregnancy, childbirth and the
puerperium
 O20-O29 Other maternal disorders predominantly related to pregnancy
 O30-O48 Maternal care related to the fetus and amniotic cavity and possible delivery problems
 O60-O77 Complications of labor and delivery
 O80, O82 Encounter for delivery
 O85-O92 Complications predominantly related to the puerperium
 O94-O9A Other obstetric conditions, not elsewhere classified

With respect to classification changes, episode of care is no longer a secondary axis of classification for
most conditions classified in Chapter 15. Instead ICD-10-CM identifies the trimester in which the
condition occurred at the fifth- and sixth-character level.

Code titles have been revised in a number of locations in Chapter 15. For instance, ICD -9-CM’s
terminology states the indication for care such as inlet contractions of pelvis (653.2). I CD-10-CM
terminology is much more descriptive of what the code represents, that is, maternal care for
disproportion due to inlet contractions of pelvis (O33.2).
Codes for elective (legal or therapeutic) abortion are classified with the abortion codes in ICD-9-CM. In
contrast, the elective abortion (without complication) code has been moved to code Z33.2, Encounter
for elective termination of pregnancy, in Chapter 21 of ICD-10-CM. Complications of induced
termination of pregnancy are found in category O04.

ICD-10-CM requires the use of a seventh character extension to identify the fetus to which certain
complication codes apply.

-Proprietary and Confidential-


©2014 himagine solutions inc. P a g e | 60
The ICD-10-CM codes for obstructed labor incorporate the reason for the obstruction into the code
therefore only one code is required rather than two as in ICD-9-CM. For example, to code obstructed
labor due to face presentation the following two ICD-9-CM codes are required: 660.0x, Obstruction
caused by malposition of fetus at onset of labor and 652.4x, Face or brow presentation. In ICD-10-CM,
only code O64.2xxx, Obstructed labor due to face presentation, is coded.

Z codes for pregnancy are for use in those circumstances when none of the problems or complications
included in the codes from the Obstetrics chapter exist (a routine prenatal visit or postpartum care).
Codes in category Z3A, weeks of gestation, may be assigned to provide additional information about the
pregnancy. The date of the admission should be used to determine weeks of gestation for impatient
admission that encompasses more than one gestational week.

Pregnancy, Childbirth, and Puerperium Guidelines


At the beginning of Chapter 15 are guidelines that provide instructions for coding professionals.
Trimesters are counted from the first day of the last menstrual period. They are defined as follows:

 1st trimester – less than 14 weeks 0 days


 2nd trimester – 14 weeks 0 days to less than 28 weeks 0 days
 3rd trimester – 28 weeks 0 days until delivery
These guidelines should be followed when selecting a code from this chapter. There are guideline
changes to a number of codes in this chapter as well. The NCHS has published chapter-specific
guidelines for Chapter 15 of ICD-10-CM:

 Guidelines I.C.15.a. General rules for obstetric cases


 Guidelines I.C.15.b. Selection of OB principal or first-listed diagnosis
 Guideline I.C.15.c. Pre-existing conditions versus conditions due to the pregnancy
 Guideline I.C.15.d. Pre-existing hypertension in pregnancy
 Guidelines I.C.15.e. Fetal conditions affecting the management of the mother
 Guideline I.C.15.f. HIV infection in pregnancy, childbirth and the puerperium
 Guideline I.C.15.g. Diabetes mellitus in pregnancy
 Guideline I.C.15.h. Long term use of insulin
 Guideline I.C.15.i. Gestational (pregnancy induced) diabetes
 Guideline I.C.15.j. Sepsis and septic shock complicating abortion, pregnancy, childbirth and the
puerperium
 Guideline I.C.15.k. Puerperal sepsis
 Guidelines I.C.15.l. Alcohol and tobacco use during pregnancy, childbirth and the puerperium

-Proprietary and Confidential-


©2014 himagine solutions inc. P a g e | 61
 Guideline I.C.15.m. Poisoning, toxic effects, adverse effects and underdosing in a pregnant
patient
 Guidelines I.C.15.n. Normal delivery, code O80
 Guidelines I.C.15.o. The peripartum and postpartum periods
 Guidelines I.C.15.p. Code O94, sequelae of complications of pregnancy, childbirth and the
puerperium
 Guidelines I.C.15.q. Abortions
 Guideline I.C.15.r. Abuse in pregnant patient

Pregnancy, Childbirth, and Puerperium Scenarios


Scenario 48 (Coding):
A 35-year-old pregnant female was admitted from birthing center for prolonged second stage of labor.
Patient is 39-weeks gestation with a history of two normal vaginal deliveries.

____________________________________________________________________________________________

Scenario 49 (Coding):
The patient, G2P1, in her 12th week of pregnancy developed severe cramping and vaginal bleeding. The
patient was subsequently taken to the emergency department and was admitted to the hospital. After
examination, the physician documented that the patient had an incomplete early spontaneous abortion.
During this pregnancy the patient had been treated for gestational hypertension of pregnancy, for which
she was monitored during this hospital stay. The patient was taken to surgery where a dilation and
curettage was performed. There were no complications following surgery. Code the diagnoses codes
only.

____________________________________________________________________________________________

Scenario 50 (Coding)
Code the following diagnosis code(s): 20-week pregnancy with low weight gain and pre-existing essential
hypertension complicating the pregnancy.

____________________________________________________________________________________________

-Proprietary and Confidential-


©2014 himagine solutions inc. P a g e | 62
Chapter 16: Certain conditions origination in the perinatal period (P00-
P96)
Chapter 16 of ICD-10-CM includes categories P00-P96 arranged in the following blocks:

 P00-P04 Newborn affected by maternal factors and by complications of pregnancy, labor and
delivery
 P05-P08 Disorders related to length of gestation and fetal growth
 P09 Abnormal findings on neonatal screening
 P10-P15 Birth trauma
 P19-P29 Respiratory and cardiovascular disorders specific to the perinatal period
 P35-P39 Infections specific to the perinatal period
 P50-P61 Hemorrhagic and hematological disorders of newborn
 P70-P74 Transitory endocrine and metabolic disorders specific to newborn
 P76-P78 Digestive system disorders of newborn
 P80-P83 Conditions involving the integument and temperature regulation of newborn
 P84 Other problems with newborn
 P90-P96 Other disorders originating in the perinatal period

A number of new subchapters have been added to Chapter 16 for certain conditions originating in the
perinatal period.
Chapter 16 of ICD-10-CM also contains terminology updates. The terms “fetus” and “newborn” used in
many ICD-9-CM code titles have been removed from ICD-10-CM code titles. Additionally, in the first
block in ICD-10-CM, newborns affected by maternal factors and by complications of pregnancy, labor,
and delivery, the phrase “suspected to be” is included in the code title as a nonessential modifier to
indicate that the codes are for use when the listed maternal condition is specified as the cause of
confirmed or suspected newborn morbidity or potential morbidity.

Some revisions to the classification have occurred as well. For instance, the subclassification for 2,500 g
and over for birth weight is no longer an option for category P05.

Perinatal (Newborn) period Guidelines


Throughout Chapter 16 are new notes that help to clarify how codes are to be used.

Codes from this chapter are only for use on the newborn or infant record, never on the maternal record
as indicated by a note that appears at the beginning of Chapter 16. Codes from this chapter are also only
applicable for liveborn infants. Further, should a condition originate in the perinatal period and continue

-Proprietary and Confidential-


©2014 himagine solutions inc. P a g e | 63
throughout the life of the child, the perinatal code should continue to be used regardless of the age of
the patient as explained by an introductory note to Chapter 16.
The NCHS has published chapter-specific guidelines for Chapter 16 of ICD-10-CM:

 Guidelines I.C.16.a. General perinatal rules


 Guideline I.C.16.b. Observation and evaluation of newborns for suspected conditions not found
 Guidelines I.C.16.c. Coding additional perinatal diagnoses
 Guideline I.C.16.d. Prematurity and fetal growth retardation
 Guideline I.C.16.e. Low birth weight and immaturity status
 Guideline I.C.16.f. Bacterial sepsis of newborn
 Guideline I.C.16.g. Stillbirth

Perinatal (Newborn) period Scenarios


Scenario 51 (Coding):
This full-term female infant was born in this hospital by vaginal delivery. Her mother has been an
alcoholic for many years and would not stop drinking during her pregnancy. The baby was born with
fetal alcohol syndrome and was placed in the NICU. What diagnosis codes are assigned?

____________________________________________________________________________________________

Scenario 52 (Coding):
Assign the code(s) for the following diagnosis: 20-day-old infant was admitted with Staphylococcus
aureus sepsis.

____________________________________________________________________________________________

Chapter 17: Congenital malformations, deformations and chromosomal


abnormalities (Q00-Q99)
Chapter 17 of ICD-10-CM includes categories Q00-Q99 arranged in the following blocks:

 Q00-Q07 Congenital malformations of the nervous system


 Q10-Q18 Congenital malformations of eye, ear, face and neck
 Q20-Q28 Congenital malformations of the circulatory system
 Q30-Q34 Congenital malformations of the respiratory system
 Q35-Q37 Cleft lip and cleft palate
 Q38-Q45 Other congenital malformations of the digestive system

-Proprietary and Confidential-


©2014 himagine solutions inc. P a g e | 64
 Q50-Q56 Congenital malformations of genital organs
 Q60-Q64 Congenital malformations of the urinary system
 Q65-Q79 Congenital malformations and deformations of the musculoskeletal system
 Q80-Q89 Other congenital malformations
 Q90-Q99 Chromosomal abnormalities, not elsewhere classified

The arrangement of ICD-10-CM’s Chapter 17 is an improvement over ICD-9-CM’s Chapter 14. Congenital
malformations, deformations, and chromosomal abnormalities have been grouped into subchapters or
blocks making it easier to identify the type of conditions classified to Chapter 17.

Congenital Malformations, Deformations, and Chromosomal Abnormalities


Guidelines
Additional guideline modifications were made to specific codes. For example, in ICD-9-CM certain
congenital malformations of the anterior segment of the eye do not have any instructions for code
usage listed below codes 743.44 or 743.45. In contrast, guidelines for code Q13.1 and Q13.81 state to
use an additional code for associated glaucoma.
Congenital anomalies or syndromes may occur as a set of symptoms or multiple malformations. If there
is no specific code, a code should be assigned for each manifestation of the syndrome, from any chapter
in the classification. For syndromes with specific codes, additional codes may be assigned to identify
manifestations not included in the specific code.
The NCHS has published chapter-specific guidelines for Chapter 17 of ICD-10-CM:

 Guideline I.C.17. Congenital malformations, deformations and chromosomal abnormalities


(Q00-Q99)

Congenital Malformations, Deformations, and Chromosomal Abnormalities


Scenarios
Scenario 53 (Coding):
Assign the code(s) for the following diagnosis: Cleft palate involving both the soft and hard palate, with
bilateral cleft lip.

____________________________________________________________________________________________

Scenario 54 (Coding):
Assign the code(s) for the following diagnosis: Penoscrotal hypospadias.

____________________________________________________________________________________________

-Proprietary and Confidential-


©2014 himagine solutions inc. P a g e | 65
Chapter 18: Symptoms, signs and abnormal clinical and laboratory
findings, not elsewhere classified (R00-R99)
Chapter 18 of ICD-10-CM includes categories R00-R99 arranged in the following blocks:

 R00-R09 Symptoms and signs involving the circulatory and respiratory systems
 R10-R19 Symptoms and signs involving the digestive system and abdomen
 R20-R23 Symptoms and signs involving the skin and subcutaneous tissue
 R25-R29 Symptoms and signs involving the nervous and musculoskeletal systems
 R30-R39 Symptoms and signs involving the genitourinary system
 R40-R46 Symptoms and signs involving cognition, perception, emotional state and behavior
 R47-R49 Symptoms and signs involving speech and voice
 R50-R69 General symptoms and signs
 R70-R79 Abnormal findings on examination of blood, without diagnosis
 R80-R82 Abnormal findings on examination of urine, without diagnosis
 R83-R89 Abnormal findings on examination of other body fluids, substances and tissues, without
diagnosis
 R90-R94 Abnormal findings on diagnostic imaging and in function studies, without diagnosis
 R97 Abnormal tumor markers
 R99 Ill-defined and unknown cause of mortality

Chapter 18 of ICD-10-CM has undergone some organizational changes. For example, in ICD-10-CM codes
for general symptoms and signs follow those related specifically to a body system or other relevant
grouping. In the comparison of Chapter 18 of ICD-10-CM to Chapter 16 of ICD-9-CM it is evident that
some codes have been moved from one chapter to another. A fairly substantial classification change
was made to hematuria. Various types of hematuria are coded in Chapter 18 unless included with the
underlying condition such as acute cystitis with hematuria. In those cases, the code is found in Chapter
14, Diseases of the genitourinary system.

Symptoms, Signs, and Abnormal Laboratory Finding, NEC Guidelines


A lengthy guideline appears at the beginning of Chapter 18 in ICD-10-CM outlining the conditions
classified to this chapter. Additionally, guidelines for code usage appear at the subchapter level.

New guidelines that clarify code usage are also found under specific codes. Code R52, pain unspecified,
includes inclusive terms and Excludes1 notes.

-Proprietary and Confidential-


©2014 himagine solutions inc. P a g e | 66
The NCHS has published chapter-specific guidelines for Chapter 18 of ICD-10-CM:

 Guideline I.C.18.a. Use of symptom codes


 Guideline I.C.18.b. Use of a symptom code with a definitive diagnosis code
 Guideline I.C.18.c. Combination codes that include symptoms
 Guideline I.C.18.d. Repeated falls
 Guideline I.C.18.e. Coma scale
 Guideline I.C.18.f. Functional quadriplegia
 Guideline I.C.18.g. SIRS due to non-infectious process
 Guideline I.C.18.h. Death NOS

Symptoms, Signs, and Abnormal Laboratory Finding, NEC Scenarios


Scenario 55 (Coding):
Assign the Glasgow coma scale code(s) when the patient had the following documented by the EMT:
Eyes do not open, no verbal response, with no motor response. The neurologist documented the
following on day 2 of the hospital admission: Eyes open to sound, verbal response produced
inappropriate words, and motor response with flexion withdrawal.

____________________________________________________________________________________________

Scenario 56 (Coding):
Assign the code(s) for the following diagnosis: Sinoatrial bradycardia.

____________________________________________________________________________________________

Chapter 19: Injury, poisoning and certain other consequences of


external causes (S00-T88)
As previously mentioned, a significant modification was made to the organization of Chapter 19. Type of
injury is the first axis of classification for the injuries in ICD-9-CM whereas specific types of injuries found
in categories S00-S99 of Chapter 19, are arranged by body region beginning with the head and
concluding with the ankle and foot. This results in the grouping of injury types together under the site
where it occurred.

 S00-S09 Injuries to the head


 S10-S19 Injuries to the neck
 S20-S29 Injuries to the thorax

-Proprietary and Confidential-


©2014 himagine solutions inc. P a g e | 67
 S30-S39 Injuries to the abdomen, lower back, lumbar spine, pelvis and external genitals
 S40-S49 Injuries to the shoulder and upper arm
 S50-S59 Injuries to the elbow and forearm
 S60-S69 Injuries to the wrist,hand and fingers
 S70-S79 Injuries to the hip and thigh
 S80-S89 Injuries to the knee and lower leg
 S90-S99 Injuries to the ankle and foot
 T07 Injuries involving multiple body regions
 T14 Injury of unspecified body region
 T15-T19 Effects of foreign body entering through natural orifice
 T20-T25 Burns and corrosions of external body surface, specified by siteT26-T28 Burns and
corrosions confined to eye and internal organsT30-T32 Burns and corrosions of multiple and
unspecified body regions
 T33-T34 Frostbite
 T36-T50 Poisoning by, adverse effect of and underdosing of drugs, medicaments and biological
substances
 T51-T65 Toxic effects of substances chiefly nonmedicinal as to source
 T66-T78 Other and unspecified effects of external causes
 T79 Certain early complications of trauma
 T80-T88 Complications of surgical and medical care, not elsewhere classified
In addition, generally the listings of conditions that follow the site are as follows:

 Superficial injury
 Open wound
 Fracture
 Dislocation and sprain
 Injury of nerves
 Injury of blood vessels
 Injury of muscle and tendon
 Crushing injury
 Traumatic amputation
 Other and unspecified injuries

Some categories in Chapter 19 have undergone title changes to reflect terminology in use today. In ICD-
10-CM, codes from blocks T20-T32 classify burns and corrosions. The addition of the term corrosion is

-Proprietary and Confidential-


©2014 himagine solutions inc. P a g e | 68
new in ICD-10-CM. The burn codes identify thermal burns, except for sunburns, that come from a heat
source. The burn codes are also for burns resulting from electricity and radiation. Corrosions are burns
due to chemicals.
A significant classification change was made to poisonings by and adverse effects of drugs,
medicaments, and biological substances (T36-T50). ICD-10-CM does not provide different category
codes to identify poisonings versus adverse effect. Instead under a single category for a specific drug are
codes for poisonings, adverse effects and underdosing of drugs, medicaments and biological substances.
Underdosing is a new terminology in ICD-10-CM and is defined as taking less of a medication than is
prescribed by a provider or the manufacturer’s instructions with a resulting negative health
consequence.

Application of 7th Characters in Chapter 19


Most categories in chapter 19 have a 7th character requirement for each applicable code. Most
categories in this chapter have three 7th character values (with the exception of fractures): A, initial
encounter, D, subsequent encounter and S, sequela. Categories for traumatic fractures have additional
7th character values.
7th character “A”, initial encounter is used while the patient is receiving active treatment for the
condition. Examples of active treatment are: surgical treatment, emergency department encounter, and
evaluation and treatment by a new physician.

7th character “D” subsequent encounter is used for encounters after the patient has received active
treatment of the condition and is receiving routine care for the condition during the healing or recovery
phase. Examples of subsequent care are: cast change or removal, removal of external or internal fixation
device, medication adjustment, other aftercare and follow up visits following treatment of the injury or
condition.

The aftercare Z codes should not be used for aftercare for conditions such as injuries or poisonings,
where 7th characters are provided to identify subsequent care. For example, for aftercare of an injury,
assign the acute injury code with the 7th character “D” (subsequent encounter).

7th character “S”, sequela, is for use for complications or conditions that arise as a direct result of a
condition, such as scar formation after a burn. The scars are sequelae of the burn. When using 7th
character “S”, it is necessary to use both the injury code that precipitated the sequela and the code for
the sequela itself. The “S” is added only to the injury code, not the sequela code. The 7th character “S”
identifies the injury responsible for the sequela. The specific type of sequela (e.g. scar) is sequenced
first, followed by the injury code.

-Proprietary and Confidential-


©2014 himagine solutions inc. P a g e | 69
Injury, Poisoning and Certain other Consequences of External Causes
Guidelines
The following guideline appears at the beginning of Chapter 19: Use secondary code(s) from Chapter 20,
External causes of morbidity, to indicate cause of injury. Codes within the T section that include the
external cause do not require an additional external cause code.

Instructions for coding open wounds have changed in ICD-10-CM. The note in ICD-9-CM defined
“complicated” used in the fourth-digit subdivisions to mean those open wounds with infection. ICD-10-
CM contains a note under the different categories for open wounds and directs the coding professional
to code also any associated wound infection.

A similar change has occurred to the instruction for complications of surgical and medical care, not
elsewhere classified (T80-T88). In ICD-9-CM, there is no guideline under this subchapter. However, ICD-
10-CM includes a note stating to use additional code (Y62-Y82) to identify devices involved and details of
circumstances.
Most categories in Chapter 19 have seventh character extensions that identify the encounter:

 A initial encounter
 D subsequent encounter
 S sequela

Additional extensions are available to identify specific encounters for fracture coding. Fracture
extensions are unique to each type of bone and type of fracture.
The NCHS has published chapter-specific guidelines for Chapter 19 of ICD-10-CM:

 Guideline I.C.19.a. Code extensions


 Guidelines I.C.19.b. Coding of injuries
 Guidelines I.C.19.c. Coding of traumatic fractures
 Guidelines I.C.19.d. Coding of burns and corrosions
 Guidelines I.C.19.e. Adverse effects, poisoning, underdosing and toxic effects
 Guideline I.C.19.f. Adult and child abuse, neglect and other maltreatment
 Guidelines I.C.19.g. Complications of care

Injury, poisoning and Certain other Consequences of External Causes


Scenarios
Scenario 57 (Coding):
This young snowboarder suffered a crash at a local ski area. He suffered multiple injuries and was
transported to this facility for treatment. He was found to have right-sided fracture of 3 ribs, and a

-Proprietary and Confidential-


©2014 himagine solutions inc. P a g e | 70
fractured carpal bone in the right wrist. None of his injuries required surgical intervention. What is
the correct diagnosis code(s)? (Do not assign external cause codes.)

____________________________________________________________________________________________

Scenario 58 (Coding):

This 85-year old patient is seen in the hospital with a diagnosis of congestive heart failure due to
hypertensive heart disease. Patient also has stage 5 chronic kidney failure. The patient had been
prescribed Lasix previously but admits that he forgets to take his medication every day. This is due to
his advanced age. What are the correct diagnosis codes? (Do not assign external codes.)

____________________________________________________________________________________________

Scenario 59 (Coding):
This patient is seen in follow-up his traumatic displaced lateral epicondyle fracture of the right elbow.
This is healing normally. What is the correct diagnosis code? (Do not assign external cause codes.)

____________________________________________________________________________________________

Scenario 60 (Coding):
This patient is seen emergently for a frontal skull fracture with a subsequent subdural hemorrhage.
There was a 45-minute loss of consciousness at the time of the accident. What is the correct diagnosis
code(s)? (Do not assign the external cause codes.)

____________________________________________________________________________________________

Chapter 20: External causes of morbidity (V00-Y99)


Chapter 20 of ICD-10-CM includes categories V00-Y99 arranged in the following blocks:

 V00-X58 Accidents
 V00-V99 Transport accidents
 V00-V09 Pedestrian injured in transport accident
 V10-V19 Pedal cycle rider injured in transport accident
 V20-V29 Motorcycle rider injured in transport accident
 V30-V39 Occupant of three-wheeled motor vehicle injured in transport accident
 V40-V49 Car occupant injured in transport accident

-Proprietary and Confidential-


©2014 himagine solutions inc. P a g e | 71
 V50-V59 Occupant of pick-up truck or van injured in transport accident
 V60-V69 Occupant of heavy transport vehicle injured in transport accident
 V70-V79 Bus occupant injured in transport accident
 V80-V89 Other land transport accidents
 V90-V94 Water transport accidents
 V95-V97 Air and space transport accidents
 V98-V99 Other and unspecified transport accidents
 W00-X58 Other external causes of accidental injury
 W00-W19 Slipping, tripping, stumbling and falls
 W20-W49 Exposure to inanimate mechanical forces
 W50-W64 Exposure to animate mechanical forces
 W65-W74 Accidental non-transport drowning and submersion
 W85-W99 Exposure to electric current, radiation and extreme ambient air temperature and
pressure
 X00-X08 Exposure to smoke, fire and flames
 X10-X19 Contact with heat and hot substances
 X30-X39 Exposure to forces of nature
 X52-X58 Accidental exposure to other specified factors
 X71-X83 Intentional self-harm
 X92-Y08 Assault
 Y21-Y33 Event of undetermined intent
 Y35-Y38 Legal intervention, operations of war, military operations and terrorism
 Y62-Y84 Complications of medical and surgical care
 Y62-Y69 Misadventures to patients during surgical and medical care
 Y70-Y82 Medical devices associated with adverse incidents in diagnostic and therapeutic use
 Y83-Y84 Surgical and other medical procedures as the cause of abnormal reaction of the patient,
or of later complication, without mention of misadventure at the time of the procedure
 Y90-Y99 Supplementary factors related to causes of morbidity classified elsewhere

As previously noted, codes for external causes are no longer found in a supplemental classification in
ICD-10-CM. The causes currently located in the ICD-9-CM E code chapter have been disseminated to
Chapter 19: Injury, poisoning and certain other consequences of external causes, or Chapter 20: External
causes of morbidity. Codes in Chapter 20 capture the cause of the injury or health condition, the intent
(unintentional or accidental; or intentional, such as suicide or assault), the place where the event

-Proprietary and Confidential-


©2014 himagine solutions inc. P a g e | 72
occurred, the activity of the patient at the time of the event, and the person’s status (namely civilian,
military).

Changes in terminology were also necessary due to the revisions made overall to this chapter. In
numerous instances, conditions included as subcategory codes in ICD-9-CM have been given a specific
category code in ICD-10-CM allowing expansion of the codes at the fourth-, fifth-, or sixth-character
level.

The external causes of morbidity codes should never be sequenced as the first-listed or principal
diagnosis. External cause codes are intended to provide data for injury research and evaluation of injury
prevention strategies. These codes capture how the injury or health condition happened (cause), the
intent (unintentional or accidental: or intentional, such as suicide or assault), the place where the event
occurred the activity of the patient at the time of the event, and the person’s status (e.g., civilian,
military).

There is no national requirement for mandatory ICD-10-CM external cause code reporting. Unless a
provider is subject to a state-based external cause code reporting mandate or there codes are required
by a particular payer, reporting of ICD-10-CM codes in Chapter 20, External Causes of Morbidity, is not
required. In the absence of a mandatory reporting requirement, providers are encouraged to voluntarily
report external cause codes, as they provide valuable data for injury research and evaluation of injury
prevention strategies.

External Causes of Comorbidity Guidelines


New notes have been added in Chapter 20 to indicate which categories require the seventh character
extensions to indicate whether the episode of care being identified was the initial, subsequent or a
secondary consequence or result (sequelae).
While there is an instructional note found at the start of category E849, place of occurrence, in ICD -9-
CM, it has been expanded in ICD-10-CM. The new guidelines state to use Y92 in conjunction with the
activity code and the place of occurrence should be recorded only at the initial encounter for treatment.
The NCHS has published chapter-specific guidelines for Chapter 20 of ICD-10-CM:

 Guidelines I.C.20.a. General external cause coding guidelines


 Guideline I.C.20.b. Place of occurrence guideline
 Guideline I.C.20.c. Activity code
 Guideline I.C.20.d. Place of occurrence, activity, and status codes used with other external cause
code
 Guideline I.C.20.e. If the reporting format limits the number of external cause codes
 Guideline I.C.20.f. Multiple external cause coding guidelines

-Proprietary and Confidential-


©2014 himagine solutions inc. P a g e | 73
 Guideline I.C.20.g. Child and adult abuse guideline
 Guideline I.C.20.h. Unknown or undetermined intent guideline
 Guidelines I.C.20.i. Late effects of external cause guidelines
 Guidelines I.C.20.j. Terrorism guidelines
 Guideline I.C.20.k. External cause status

External Causes of Comorbidity Scenarios


Scenario 61 (Coding):
12-year-old female is seen for continued pain related to her elbow fracture. Six weeks ago, this patient
injured her elbow when she fell while skating at the local roller rink. After further evaluation, the
attending physician found a nonunion of the previously displaced right distal humerus fracture. She will
be scheduled for surgery in the next two days. What diagnosis codes are assigned?

____________________________________________________________________________________________

Scenario 62 (Coding):
This patient is a 20 year old male who is brought to the emergency department by ambulance, found to
be the victim of a random beating. This patient was walking in his neighborhood park when he was
pulled down and then beaten during a fight. The patient was comatose when found by the paramedics
but did open his eyes in response to pain; however, has no verbal or motor response. The patient was in a
coma upon admission but regained consciousness within 40 minutes of arriving in the ED, less than an
hour after being found. The MRI is negative for fractures or internal bleeding. The physician describes the
injury as a closed head injury with loss of consciousness of less than 1 hour. What diagnosis codes are
assigned?

____________________________________________________________________________________________

Chapter 21: Factors influencing health status and contact with health
services (Z00-Z99)
Coding professionals will find the listing of codes for factors influenci ng health status and contact with
health services a bit different in ICD-10-CM than what is currently found in ICD-9-CM. The following
blocks represent the ICD-10-CM arrangement:

 Z00-Z13 Persons encountering health services for examination and investigation


 Z14-Z15 Genetic carrier and genetic susceptibility to disease
 Z16 Resistance to antimicrobial drugs
 Z17 Estrogen receptor status

-Proprietary and Confidential-


©2014 himagine solutions inc. P a g e | 74
 Z18 Retained foreign body fragments
 Z20-Z28 Persons with potential health hazards related to communicable diseases
 Z30-Z39 Persons encountering health services in circumstances related to reproduction Z40-Z53
Encounters for other specific health care
 Z55-Z65 Persons with potential health hazards related to socioeconomic and psychosocial
circumstances
 Z66 Do not resuscitate status
 Z67 Blood type
 Z68 Body mass index (BMI)
 Z69-Z76 Persons encountering health services in other circumstances
 Z77-Z99 Persons with potential health hazards related to family and personal history and certain
conditions influencing health status
Some categories in Chapter 21 have rephrased titles to better reflect the situations the codes classify.
Some conditions in ICD-10-CM no longer have the specificity that they did in ICD-9-CM. For instance,
ICD-9-CM category V09 had subclassifications to identify the specific drug in which the microorganism
was resistant. This is not the case in ICD-10-CM where there is only one category, Z16, to represent
infection with drug-resistant microorganisms. Another example of decreased specificity in ICD-10-CM is
code Z23, Encounter for immunization. This code is not further classified. In ICD-9-CM, category codes
V03, V04, V05, and V06 are used to identify the types of immunizations.

Factors Influencing Health Status and Contact with Health Services Guidelines
The note at the beginning of this chapter has been modified from what it states in ICD -9-CM. All codes in
Chapter 21 are affected by these revised guidelines.
Instructional notes also have been added to different categories to explain how codes should be
assigned.
The NCHS has published chapter-specific guidelines for Chapter 21 of ICD-10-CM:

 Guideline I.C.21.a. Use of Z codes in any healthcare setting


 Guideline I.C.21.b. Z codes indicate a reason for an encounter
 Guidelines I.C.21.c. Categories of Z codes

Factors Influencing Health Status and Contact with Health Services Scenarios
Scenario 63 (Coding):
Assign the code(s) for the following diagnosis: Medical examination of four-year-old child prior to
admission to preschool.

____________________________________________________________________________________________

-Proprietary and Confidential-


©2014 himagine solutions inc. P a g e | 75
Scenario 64 (Coding):
Assign the code(s) for the following encounter: Patient seen for fitting of right artificial leg after patient
had below-knee amputation due to medical condition.

____________________________________________________________________________________________

-Proprietary and Confidential-


©2014 himagine solutions inc. P a g e | 76
Overview of ICD-10-CM
Classification Changes by Body
System/Chapter (Not all Inclusive)

-Proprietary and Confidential-


©2014 himagine solutions inc. P a g e | 77
Overview of ICD-10-CM Classification Changes by Body
System/Chapter (Not all Inclusive)
Circulatory
 Cerebral infarctions and cerebral bleeds in ICD-10-CM now classify specific artery involvement
for appropriate classification

 CHF classification is a combination code enabling diastolic/systolic heart failure to only require
one (1) code

 AMIs acute timeframe is changed to four (4) weeks in ICD-10-CM

 STEMI AMI codes in ICD-10-CM now classify specific artery involvement for appropriate
classification

 Classification now exists for subsequent AMIs within 4 weeks of the initial AMI

 Subsequent AMIs require both a code from I21 and I22 category to appropriately classify disease
process

 Complications of AMIs are specifically classified in ICD-10-CM. In ICD-9-CM these conditions


were classified to other and acquired conditions

 When the patient requires continued care for the myocardial infarction, codes from category I21
may continue to be reported for the duration of 4 weeks (28 days) or less from onset, regardless
of the healthcare setting, including when a patient is transferred from the acute care setting to
the post-acute care setting if the patient is still within the four weeks time frame in ICD-10-CM,
this coding guideline is different from coding guidelines in ICD-9-CM

Respiratory
 “Acute” Respiratory Failure – documentation must reflect “acute” – whereas in ICD-9-CM,
unspecified respiratory failure defaulted to “acute respiratory failure”, in ICD-10-CM, it does not
default, therefore documentation of “acute” is required when diagnosing respiratory failure if
appropriate.

 Tonsillitis is subclassified in ICD-10-CM into recurrent and unspecified as well as identifying


specific organism(s)

 Acute bronchitis category is greatly expanded with organism specificity

 Human metapneumovirus is classified as a specific organism in some disease processes in the


respiratory system

-Proprietary and Confidential-


©2014 himagine solutions inc. P a g e | 78
 Influenza now subclassified with gastrointestinal manifestations as well as myocarditis, otitis
media, encephalopathy and other manifestations

 Emphysema is further subdivided to identify unilateral pulmonary emphysema panlobular and


centrilobular

 Asthma is further subdivided to identify Mild intermittent/perisistent, moderate and severe


persistent, exercise induced bronchospasm as well as cough variant asthma

 COPD and bronchiectasis are further classified to “with lower respiratory infection” and “acute
exacerbation”

 Pneumoconiosis due to other inorganic dusts category greatly expanded to classify specific
inorganic dusts

 H1N1 influenza and avian influenza are expanded to delineate gastrointestinal manifestations
as well as myocarditis, otitis media, encephalopathy and other manifestations

 Instructional notes are more numerous in ICD-10-CM for this chapter than in ICD-9-CM. Coders
will need to pay close attention to instructions for accurate classification of respiratory diseases

Nervous System
 Alzheimer’s disease is further classified to early and late onsets

 Secondary Parkinsonism (G21) is further classified to Malignant neuroleptic syndrome, Other


drug induced secondary parkinsonism (Neuroleptic induced parkinsonism, Other drug induced
secondary parkinsonism), Secondary parkinsonism due to other external agents,
Postencephalitic parkinsonism, Vascular parkinsonism and Other/unspecified

 Other extrapyramidal and movement disorders (G25.0-G25.6),choreas, dystonias, tics and


tremors are further classified to drug-induced conditions

 Hereditary ataxia (G11) further sub-divided to delineate Congenital nonprogressive ataxia, Early
onset cerebellar ataxia, Late onset cerebellar ataxia, and Cerebellar ataxia with defective DNA
repair and other/unspecified

 Disorders of autonomic nervous system (G90) delineates new category for Complex regional
pain syndrome I (CRPS I) further classifying upper/lower limb and laterality

 Cluster, vascular, tension, drug-induced headaches and other trigeminal autonomic cephalgias
(TAC) (G44.0) further classify with/without intractable headache

 Other demyelinating diseases of CNS (G37) includes further subclassifications to delineate:


Diffuse sclerosis of CNS, Central demyelination of corpus callosum, Central pontine myelinolysis,

-Proprietary and Confidential-


©2014 himagine solutions inc. P a g e | 79
Acute transverse myelitis, Subacute necrotizing myelitis of CNS and Concentric sclerosis [Balo] of
CNS

 Hemiplegia and Hemiparesis (G81) as well as monoplegia are further classified to laterality and
dominant/non-dominant sides

 Other specified paralytic syndromes (G83) are further subclassified to: Brown-Sequard,
anterior/posterior cord and Todd’s paralysis (postepileptic)

 Migraines (G43) are further classified to delineate: Cycling vomiting, Opthalmoplegic migraine,
Periodic headache syndromes in child or adult and Menstrual migraine

 Other and unspecified myopathies (G72) are further classified to Drug induced and Alcoholic

 Other and unspecified polyneuropathies (G62) are further subdivided to delineate if due to
Critical illness or is Radiation-induced

 Vascular syndromes of brain in cerebrovascular diseases (G46) is a new category to ICD-10-CM


and includes sub-classifications for: Middle cerebral artery syndrome, Anterior/posterior
cerebral artery syndrome, Brain stem stroke syndrome, Cerebellar stroke syndrome, Pure
motor/sensory lacunar syndrome, Other lacunar syndromes and Other vascular syndromes of
brain in cerebrovascular disease

 Toxic encephalopathy (G92) has its own classification in ICD-10-CM

Endocrine System
 Iodine-deficiency thyroid disorders are new classifications in ICD-10-CM and are delineated as
congenital, related thyroid disorders (goiters) and subclinical hypothyroidism due to iodine
deficiency

 Congenital hypothyroidism is expanded to include with/without goiter

 Acquired hypothyroidism now specifies drug-induced, postinfectious, atrophy and myxedema


coma

 Poorly controlled, inadequately controlled and out of control diabetes mellitus are classified to
Diabetes, by type, with hyperglycemia – this is a new classification to ICD-10-CM

 Gestational diabetes is classified to insulin controlled or diet controlled and puerperal in the OB
chapter

 “Other” specified complications of diabetes mellitus are expanded to include: Neuropathic


arthropathy, dermatitis, oral complications including periodontal disease and other oral disease.
Documentation of complication relationship to diabetes continues to be an opportunity for
improvement

-Proprietary and Confidential-


©2014 himagine solutions inc. P a g e | 80
 Secondary diabetes mellitus is now classified to “due to underlying condition and drug or
chemical induced”. Provider documentation must reflect underlying cause of DM

 There is no longer a specific classification for “uncontrolled” vs. “controlled” status

 Cushing’s syndrome is further classified to Pituitary-dependent Cushing’s, Nelson’s syndrome,


Drug-induced Cushing’s syndrome, Ectopic ACTH syndrome, Alcohol-induced pseudo-Cushing’s
syndrome

 Obesity is further classified to morbid/other obesity obesity due to hyperalimentation as well as


drug-induced

 Megavitamin-B6 syndrome is now classified in ICD-10-CM and is included under obesity


disorders

Blood and Blood-Forming Organs


 B-12 deficiency anemia (D51) is expanded to include due to intrinsic factor deficiency, due to
selective Vitamin B12 malabsorption with proteinuria, transcobalamin II deficiency and
other/unspecified

 Folate deficiency anemia (D52) is expanded to include dietary, drug-induced and


other/unspecified

 Hemolytic anemias due to enzyme deficiencies (D55) is expanded to include: Glucose-6-


phosphate dehydrogenase (G6PD) deficiency, Other disorders of glutathione metabolism,
Glycolytic enzymes, Nucleotide metabolism and other/unspecified enzyme disorders

 Auto-immune and non-autoimmune anemias (D59) are further subclassified to specify drug-
induced and paroxysmal nocturnal hemoglobinuria (Marchiafava-Micheli) types as well as other
nonautoimmune hemolytic anemias

 Aplastic anemia (D61) is further subclassified to drug-induced, due to other external agents,
idiopathic and other

 Sideroblastic anemia (D64) is expanded to classify Hereditary, secondary due to disease,


secondary due to drugs and toxins and other as well as congenital dyserythropoietic

 Other coagulation defects (D68) is expanded to include Hemorrhagic disorder due to extrinsic
circulating anticoagulants (intrinsic already classified in ICD-9-CM) as well as Primary and other
thrombophilia

 Diseases of the spleen (D73) are further subdivided to include Hyposplenism, abscess, cyst and
infarction

 Methemoglobinemia (D74) is further subdivided into congenital and other/unspecified

-Proprietary and Confidential-


©2014 himagine solutions inc. P a g e | 81
 Immune disorders are almost completely revised in ICD-10-CM with significant expansion of
immune disease classifications

 Sarcoidosis (D86) is significantly expanded to include specific organ systems rather than
requiring an additional code to signify “in diseases classified elsewhere” in each chapter.
Combination codes for sarcoidosis include: Lung, lymph nodes, lung with lymph nodes,
iridocyclitis, pyelonephritis, myocarditis, arthropathy, myositis and other sites)

Infectious Diseases
 A second code from subcategory R65.2, Severe sepsis, should not be assigned unless severe
sepsis or an associated acute organ dysfunction is documented

 The acute organ dysfunction must be documented by the physician as “due to” the sepsis to be
considered a manifestation of the sepsis thereby qualifying as a “severe” sepsis with organ
dysfunction

 Only one code is required to classify sepsis alone with no organ dysfunction or documentation
of severe sepsis

 R65.21, Severe sepsis with septic shock, must be assigned if septic shock is documented in the
medical record, even if the term severe sepsis is not documented. Severe sepsis is assumed
when shock is present.

 The term “urosepsis” is nonspecific and is not synonymous with sepsis. There is not a default
code in the Alphabetic Index in ICD-10-CM. Should a provider use the term “urosepsis”, he/she
must be queried for clarification of specific disease process.

 Disease continuum for sepsis is: bacteremia, septicemia, sepsis, severe sepsis, sepsis with septic
shock and MODS (Multiple Organ Dysfunction Syndrome)

 Two (2) common clinical laboratory findings indicating severe sepsis are Hyperlactatemia and
lactic acidosis

 Many infectious diseases are further subdivided to specify body system or specific disease
process (i.e., meningitis, peritonitis, etc.) (see Typhoid fever (A01) for an example)

 Tuberculosis no longer has a 5th digit subclassification for bacterial or histological findings

 Listeriosis is greatly expanded in ICD-10-CM delineating specific body sites for infection. ICD-9-
CM only had 1 code classification for this disease

 Whooping cough is subclassified to delineate with and without pneumonia

 Nonpneumonic Legionnaires’ disease [Pontiac fever] is classified in ICD-10-CM but no


corresponding classification exists in ICD-9-CM

-Proprietary and Confidential-


©2014 himagine solutions inc. P a g e | 82
 Brazilian purpuric fever is classified in ICD-10-CM with no corresponding classification existing in
ICD-9-CM

OB-GYN
 Fistulae involving female genital tract classification (N82) is expanded to include Vesicovaginal,
Other urinary-genital tract fistulae, Vagina to small intestine, Vagina to large intestine and Other
intestinal-genital fistulae

 Complications associated with artificial fertilization (N98) classification is created in ICD-10-CM


with the following complications specified – Infection associated with artificial insemination,
Hyperstimulation of ovaries, Complications of attempted introduction of fertilized ovum
following in vitro fertilization, Complications of attempted introduction of embryo in embryo
transfer, Other/unspecified

 Menstruation disorders are expanded to include Primary/secondary amenorrhea and


oligomenorrhea; in addition, excessive and frequent menstruation with regular/irregular cycle
and at puberty, Ovulation bleeding and excessive bleeding in premenopausal period is also
classified in ICD-10-CM

 Intraoperative and postprocedural complications and disorders of OB-GYN system, are classified
including residual ovary syndrome and post-procedural adhesions of vagina and peritoneum

 OB conditions are classified by trimester – 1st trimester (< 14 weeks 0 days), 2nd trimester (14
weeks 0 days to < 28 weeks 0 days) and 3rd trimester (28 weeks 0 days until delivery)

 Gestational diabetes is classified to insulin controlled or diet controlled

 Complications of labor affecting fetus must specify which fetus is affected in those instances
with multiple gestation (i.e., fetus 1,2,3,4,5 or other fetus (when number of fetuses exceeds
quintuplets))

 Hydatidiform mole further classified to classical and incomplete/partial

 Abortion and ectopic/molar pregnancies with complications are further classified to with cardiac
arrest, sepsis, and UTI

 Classifications are added to OB chapter to delineate malignant neoplasm complicating


pregnancy with trimester specification as well as injuries or poisonings occurring during
pregnancy

 Genitourinary infections affecting pregnancy are classified by specific site, i.e., kidney, bladder,
urethra, cervix, salpingo-oophoritis and other/unspecified sites

 Diabetes mellitus requires specificity of type – Type 1 and Type 2

-Proprietary and Confidential-


©2014 himagine solutions inc. P a g e | 83
 Complications of anesthesia during pregnancy are specifically classified and require detailed
documentation of type of complication for classification of disorder as well as documentation of
relationship to anesthesia

 Underlying causes of cephalo-pelvic disproportion and other obstructed labor have been
expanded to include both fetal and maternal abnormalities. Documentation of specific
underlying causes is necessary for appropriate classification of these conditions and for accurate
reflection of patient acuity and outcome

 Physical, sexual and psychological abuse are classified complicating pregnancy, childbirth and
puerperium with further expansion to delineate trimester of occurrence

Urinary System
 Acute and chronic glomerulonephritis/nephrotic/nephritic syndrome categories (N00-N08) have
been significantly revised and expanded in ICD-10-CM to include greater specificity related to
glomerular lesions and glomerulonephritis

 Recurrent and persistent hematuria (N02) is now classified in ICD-10-CM with expansion to
include many different types of glomerulonephritis; ICD-9-CM only classified hematuria to
“gross”, “microscopic” or “unspecified”. If hematuria is unspecified it is classified to the Signs
and Symptoms chapter in ICD-10-CM

 New classifications for “Hereditary nephropathy” and “Isolated proteinuria with specified
morphological lesion” (N06-N07) are included in ICD-10-CM

 Classification for hydronephrosis (N13) is expanded to include Hydronephrosis with ureteral


stricture and Hydronephrosis with renal and ureteral calculous obstruction and
other/unspecified

 Classifications for Drug and heavy-metal-induced tubulo-interstitial and tubular conditions as


well as other renal tubulo-interstitial conditions are created with several different types of
nephropathy and other renal conditions classified in this disease process

 Cystitis classification is expanded to include with/without hematuria

 Neuromuscular dysfunction of bladder, NEC is created to include Uninhibited, Reflex and Flaccid
neuropathic bladder disorders

 Post-traumatic urethral stricture (N35) is expanded to include male (Meatal, Bulbous,


Membranous, Anterior, and Unspecified) and female (Due to childbirth and Other)

 Postprocedural/post-infective urethral strictures classifications are expanded to include male


(Meatal, Bulbous, Membranous, Anterior, and Unspecified) and female

 Acute/chronic prostatitis classification is expanded to include with/without hematuria

-Proprietary and Confidential-


©2014 himagine solutions inc. P a g e | 84
 Spermatocele classification is expanded to include single, multiple or unspecified

 Disorders of prepuce (N47) is greatly expanded to include Adherent prepuce, newborn,


Phimosis, Paraphimosis, Deficient foreskin, Benign cyst of prepuce, Adhesions of prepuce and
glans penis, Balanoposthitis, Other inflammatory diseases of prepuce and Other disorders of
prepuce

 Azoospermia/Oligospermia classifications (N46) are greatly expanded to include organic, due to


drug therapy, infection, obstruction, radiation, systemic disease and other extratesticular
disease

 Priapism (N48) classification is expanded to include underlying causes of Unspecified/other


cause, Due to trauma, Due to disease classified elsewhere and Drug-induced
 Male erectile dysfunction (N52) is greatly expanded to include vasculogenic, drug induced, post -
surgical (following radical prostatectomy/cystectomy, urethral or post simple prostatectomy)
and other /unspecified causes
 Ejaculatory dysfunction (N53) classification is expanded to include Retarded, Painful,
Retrograde, Anejaculatory orgasm, and Other

-Proprietary and Confidential-


©2014 himagine solutions inc. P a g e | 85
Case Studies

-Proprietary and Confidential-


©2014 himagine solutions inc. P a g e | 86
Case Studies

NOTE: Please code ONLY the diagnosis codes in the Case Studies
listed below.
Respiratory Disease
Scenario 1 (Coding):
Diagnosis and Procedures: Patient is a 70 year old male admitted to ER short of breath and
dusky appearance. He reports coughing almost non-stop for the last 3 days and is
experiencing severe chest pain. He has a history of COPD. Copious sputum is produced and
cultured. Patient is admitted to the hospital for treatment of bronchitis and respiratory
treatments for severe shortness of breath with a RR 30, PCO2 60, pH 7.29 and hypoxemia.
Patient is placed on a ventilator 6 hours later for 72 hours. Sputum cultures return positive
for Mycoplasma pneumoniae and patient is prescribed azithromycin. With appropriate
documentation of Acute Respiratory Failure, Acute Mycoplasma Bronchitis and ventilator
how would this case be coded?

____________________________________________________________________________________________

___________________________________________________________________________________________

Scenario 2 (Coding):
Diagnosis and Procedures: Patient undergoes a total sigmoidectomy for CA of sigmoid colon
and liver/lymph node biopsies. Patient develops acute pneumothorax 1 day after the
procedure. A chest tube is inserted in the right upper lobe of lung. Pathology report also notes
metastasis to the liver and intestinal lymph nodes. The physician documents CA colon with a
removal of the sigmoid colon and lymph nodes and biopsy of the liver. With appropriate
documentation of diagnoses (CA sigmoid colon, mets to liver/lymph nodes and postprocedura l
pneumothorax) what codes would be utilized?

-Proprietary and Confidential-


©2014 himagine solutions inc. P a g e | 87
____________________________________________________________________________________________

____________________________________________________________________________________________

Scenario 3 (Coding):
Diagnosis: Patient is admitted with cough, sore throat, fever and extreme fatigue with
inability to take a deep breath and shortness of breath. He has a history of rheumatoid
arthritis in both hands for which he has been treated with methotrexate after a flare of the
arthritis. CXR demonstrates an infiltrate in the right middle lobe and diffuse linear opacities.
Patient is admitted to the hospital for treatment. Patient is diagnosed with Interstitial Lung
Pneumonia with no further specifics. With appropriate documentation, how is RA as the
underlying cause of the interstitial pneumonia coded? How would you code the condition if
the medication treatment [Methotrexate] was the underlying cause and it was documented by
the physician as acute drug-induced interstitial pneumonia?

____________________________________________________________________________________________

____________________________________________________________________________________________

Scenario 4 (Coding):
Diagnosis: Patient is admitted with cough, sore throat, fever and extreme fatigue with
difficulty breathing and shortness of breath. Patient is also extremely nauseous with vomiting
and diarrhea the last 12 hours. Patient appears to be dehydrated with dry mouth and poor
skin turgor. CXR demonstrates an infiltrate in the right middle lobe. Patient is admitted to the
hospital for treatment with IV fluids, antiemetic and antibiotics and antivirals for infiltrate.
Patient is diagnosed with Influenza with no further specifics. With appropriate
documentation of influenza related N&V&D as well as confirmation of infiltrate on CXR as
being influenza pneumonia what codes would be assigned? Patient is also diagnosed with
dehydration.

____________________________________________________________________________________________

-Proprietary and Confidential-


©2014 himagine solutions inc. P a g e | 88
____________________________________________________________________________________________

Circulatory Disease
Scenario 1 (Coding):
Diagnosis and Procedure: The patient is a 62-year old female who was admitted to another
hospital on 1/8/XX after experiencing tachycardia. There she underwent a cardiac
catheterization, showing the presence of severe two-vessel coronary artery disease. The
patient does not have any history of a CABG in the past. The patient has a history of sick sinus
syndrome, hypertensive heart disease, and CHF. She was transferred to our hospital to
undergo a percutaneous transluminal angioplasty.

Physical Examination: No physical abnormalities were found on the cardiovascular


examination. Pulse 50 and BP 100/85. HEENT: PERRLA, faint carotid bruits. Lungs: clear to
percussion and auscultation. Heart: Normal sinus rhythm with a 2.6 systolic ejection murmur.
Extremities and abdomen were negative.

Hospital Course: To manage the patient’s sick sinus syndrome, a permanent dual chamber
pacemaker was implanted on 1/9/XX with leads inserted in right atrium and right ventricle.
On 1/10/XX the patient underwent a PTCA of both the left anterior descending artery and the
right coronary artery (with DES) without complications and good results obtained.

Postoperatively, the patient was stable and was subsequently discharged. The patient’s
hypertensive heart disease and CHF were managed and monitored during the hospital stay
and the patient continued taking her normal medications for these conditions.

____________________________________________________________________________________________

____________________________________________________________________________________________

-Proprietary and Confidential-


©2014 himagine solutions inc. P a g e | 89
Scenario 2 (Coding):
Diagnosis and Procedure:

A patient was admitted through the emergency room complaining of chest pain with
radiation down the left arm increasing in severity over the past three hours. Initial impression
was impending myocardial infarction, and the patient was taken dire ctly to the surgical suite,
where percutaneous transluminal angioplasty with insertion of coronary stent was carried
out on the right coronary artery. Infarction was aborted, and the diagnosis was listed as acute
coronary insufficiency.

____________________________________________________________________________________________

____________________________________________________________________________________________

Scenario 3 (Coding):
Diagnosis: A patient was admitted with systolic heart failure, acute on chronic, congestive
heart failure, and unstable angina. The unstable angina was treated with nitrates, and IV
Lasix was administered to manage the heart failure. Both conditions improved, and the
patient was discharged to be followed on an outpatient basis.

____________________________________________________________________________________________

____________________________________________________________________________________________

Scenario 4 (Coding):
Diagnosis and Procedure: The reason for this woman’s admission was for a repair of a 4.7
centimeter infrarenal abdominal aortic aneurysm. She also had arterial hypertension.
Because of her strong family history of aneurysms she wished to have her aneurysm removed
on an elective basis rather than waiting for it to follow its natural course. An infrarenal
excision of the aortic aneurysm, open approach, was performed using a 16 millimeter Dacron

-Proprietary and Confidential-


©2014 himagine solutions inc. P a g e | 90
graft replacement. The procedure was successful and the patient was discharged on the fifth
postoperative day.

____________________________________________________________________________________________

____________________________________________________________________________________________

Scenario 5 (Coding):
Diagnosis and Procedure: This patient was admitted for repair of a left common carotid
stenosis. Four months earlier she had suffered a cerebral hemorrhage that resulted in apraxia
which required additional nursing assistance. The open left endarterectomy was successfully
accomplished and the patient was discharged on the fourth hospital day.

____________________________________________________________________________________________

____________________________________________________________________________________________

Endocrine Disease
Scenario 1 (Coding):
Diagnosis: The patient is being seen because of increasingly irritating symptomatology,
including nervousness, irritability, increased perspiration, shakiness and increased appetite
with unexplained weight loss, increased heart rate, palpitations, and sleeping difficulties. A
thyroid stimulating hormone test revealed elevated levels and a thyroid nuclear medicine
scan revealed hyperactivity of the entire thyroid gland. Based on the diagnostic findings the
patient was diagnosed with hyperthyroidism with multinodular goiter. The patient was
started on oral anti-thyroid medication. Arrangements were also made for patient to see a
cardiologist due to the fact her palpitations were more pronounced than seen in other
patients with hyperthyroidism. What diagnosis codes are assigned?

____________________________________________________________________________________________

____________________________________________________________________________________________

-Proprietary and Confidential-


©2014 himagine solutions inc. P a g e | 91
Scenario 2 (Coding):
Diagnosis and Procedure: This morbidly obese patient with a BMI of 39 was admitted to have
a laparoscopic gastroplasty with gastric banding restriction.

____________________________________________________________________________________________

____________________________________________________________________________________________

Scenario 3 (Coding):
Diagnosis: A 35 year old woman was having flushing and sleeplessness due to premature
menopause.

____________________________________________________________________________________________

____________________________________________________________________________________________

Scenario 4 (Coding):
Diagnosis: Patient with type 2 diabetes mellitus participated in a strenuous game of
racquetball without adjusting his insulin dosage; he is admitted with blood sugar of 35 and is
diagnosed as being hypoglycemic.

____________________________________________________________________________________________

____________________________________________________________________________________________

Scenario 5 (Coding):

Diagnosis: The patient was seen with severe abdominal cramping, nausea and vomiting, and
diarrhea. She stated she ate turkey salad several hours before these symptoms developed. The
patient was treated with IV therapy. Diagnosis: Dehydration and Salmonella gastroenteritis.

____________________________________________________________________________________________

__________________________________________________________________________ __________________

-Proprietary and Confidential-


©2014 himagine solutions inc. P a g e | 92
Blood and Blood Forming
Scenario/Exercise 1:
Match the disease process to the definition:

A. ____ Anemia ____ A deficiency in platelets, the cells that are important in
B. ____ Thrombocytopenia blood clotting
C. ____ Sickle-Cell Anemia
D. ____ Aplastic Anemia
____ A condition in which there is a deficiency of red blood
E. ____ Sickle Cell Trait
F. ____ Pancytopenia cells because the bone marrow is failing to produce them

____ A condition in which blood is deficient in the amount of


hemoglobin in red blood cells or in the volume of red blood
cells

____ A condition that occurs when a child receives the trait


from only one parent

____ A type of aplastic anemia in which red blood cells, white


blood cells, and platelets are all deficient

____ A hereditary disease of the red blood cells passed to a


child when both parents carry the genetic trait

-Proprietary and Confidential-


©2014 himagine solutions inc. P a g e | 93
Scenario 2 (Coding):

Diagnosis and Procedure: A patient six months pregnant was diagnosed as having an iron-
deficiency anemia and was admitted for transfusion of nonautologous packed red blood cells,
via peripheral vein.

____________________________________________________________________________________________

____________________________________________________________________________________________

Scenario 3 (Coding):
Diagnosis: A patient with sickle-cell anemia presents to the emergency department with a two
to three day history of severe right leg and arm pain. After she was admitted parenteral
narcotics were administered and the pain improved. The blood counts returned to a stable
level within 24 hours. The patient was diagnosed with Sickle-cell pain crisis.

____________________________________________________________________________________________

____________________________________________________________________________________________

Scenario 4 (Coding):
Diagnosis: The patient was diagnosed with Coombs’ negative hemolytic anemia four years
earlier. Since the diagnosis her disease course waxed and waned. During some bouts she had
15 to 20 blood transfusions of two to three units of packed red blood cells each. This
admission was for splenectomy for hypersplenism. . A total open splenectomy was performed
without incident. Her postoperative recovery was without incident. The patients discharge
diagnosis was Hypersplenism secondary to acquired hemolytic anemia.

____________________________________________________________________________________________

____________________________________________________________________________________________

-Proprietary and Confidential-


©2014 himagine solutions inc. P a g e | 94
Urinary/Kidney Disease
Scenario 1 (Coding):
Diagnosis and Procedure: Patient is a 13 year old male admitted to ER with hematuria and
generalized swelling, lethargy and fatigue. Urine analysis shows proteinuria and a kidney
biopsy is scheduled for suspected glomerulonephritis. A percutaneous biopsy of the left kidney
returns positive for acute DDD (Dense Deposit Disease) or membranoproliferative
glomerulonephritis type II (MPGN II). Physician (provider) documents diagnosis of
glomerulonephritis with no further specificity. With the appropriate documentation of acute
glomerulonephritis due to DDD what diagnosis and procedure codes would be utilized?

____________________________________________________________________________________________

____________________________________________________________________________________________

Scenario 2 (Coding):
Diagnosis: Patient is admitted with a history of hypertension and Type 2 diabetes with
generalized fatigue, swelling of legs, decreased appetite, infrequent urination and nausea and
vomiting. Patient has been diagnosed with chronic kidney disease (CKD) in the past. G FR is 20.
He is treated with IV Furosemide 60 mg per day X 2 days and placed on a maintenance dose
upon discharge. He is advised to follow a reduced sodium diet and decrease protein intake.
With the appropriate documentation of CKD related to DM and the stage 4 of the CKD, what
codes would be required?

____________________________________________________________________________________________

____________________________________________________________________________________________

Scenario 3 (Coding):
Diagnosis and procedures: Patient is admitted with severe flank pain, painful urination, fever
and gross hematuria. Patient is also extremely nauseous with vomiting the last 12 hours. IVP

-Proprietary and Confidential-


©2014 himagine solutions inc. P a g e | 95
with high osmolar contrast shows hydronephrosis on the left side and CT scan with low
osmolar contrast reveals ureteral calculus. Physician (provider) diagnoses a ureteral calculus
with no further specification. With appropriate documentation of ureteral calculus with
hydronephrosis, what diagnoses and procedures would be coded?

____________________________________________________________________________________________

____________________________________________________________________________________________

Nervous System
Scenario 1 (Coding):
Diagnosis: Patient is a 70 year old male admitted to ER after being found by police wandering
the street, mentally confused and combative. Police reported the patient was verbally abusive
and had tried to assault a passerby on the street. Police also reported the patient had been
reported missing from a nursing home that morning. ED physician documents diagnosis of
Alzheimer’s disease with no further specificity. What is required for accurate classification of
Alzheimer’s disease? With appropriate documentation of late onset Alzheimer’s with
dementia and behavioral disturbances what codes would be assigned?

____________________________________________________________________________________________

____________________________________________________________________________________ ________

Scenario 2 (Coding):
Diagnosis: Patient is admitted with upper limb tremors after being placed on perphenazine
for mood disorder. His medication was changed to quetiapine, but tremor was unabated.
Motor, sensory, and cerebellar testing were normal. Other findings included mild stoop with
normal gait and reflexes. He demonstrated a mild chin tremor and moderately irregular
finger tapping bilaterally. A diagnosis of Parkinsonism is documented by the attending

-Proprietary and Confidential-


©2014 himagine solutions inc. P a g e | 96
physician. With appropriate documentation of Secondary Parkinsonism due to adverse effect
of perphenazine, what code(s) would be assigned?

____________________________________________________________________________________________

____________________________________________________________________________________________

Scenario 3 (Coding):
Diagnosis: A 75 year old male is admitted with a cerebral infarction. He is demonstrating
expressive aphasia and apraxia. Physician documents cerebral infarction as the final
diagnosis. Patient is discharged to a rehab facility. With appropriate documentation of
cerebral infarction (thrombosis) on the left side with middle cerebral artery involvement
what would be the code(s) assigned to this case?

____________________________________________________________________________________________

____________________________________________________________________________________________

OB/GYN
Scenario 1 (Coding):
Diagnosis and Procedure: Physician documents diagnosis of vaginal fistula with repair with
no further specificity. A repair is performed with a resection of a portion of the small intestine
with anastomosis of small to small intestine and a repair of the vaginal wall with mesh. Path
report reveals a fistula involving vagina to small intestine. With appropriate documentation
of vaginal fistula involving small intestine with repair, what codes would be applied?

____________________________________________________________________________________________

____________________________________________________________________________________________

-Proprietary and Confidential-


©2014 himagine solutions inc. P a g e | 97
Scenario 2 (Coding):
Diagnosis: Patient is admitted with a history of artificial fertilization 3 days prior. She is
exhibiting a fever of 101 degrees, chills and shaking. She complains she has been running a
low grade fever for the last 36 hours with a sudden escalation to 101 degrees the morning of
admission. Physician (provider) documents endometritis with no further specification. With
appropriate documentation of acute endometritis due to artificial insemination procedure,
what code(s) would be assigned?

____________________________________________________________________________________________

____________________________________________________________________________________________

Scenario 3 (Coding):
Diagnosis and Procedure: Patient with a history of total abdominal hysterectomy 12 months
ago is now admitted for repair of vaginal vault prolapse via vaginal approach with removal of
some of the vaginal tissue and inserting mesh to reinforce it. With appropriate documentation
of hysterectomy related vaginal vault prolapsed, what code(s) would applied for both the
diagnosis and procedure?

____________________________________________________________________________________________

____________________________________________________________________________________________

Infection Disease
Scenario 1 (Coding):
Diagnosis: Patient is admitted to the hospital and taken to the ICU with a T 102, Pulse 120, RR
25, altered mental status, BP 85/62, confusion and WBCs 25.2. He is treated with IV
antibiotics. Physician documents Urosepsis as the principal diagnosis. Can this case be coded
as it is documented? Why?

-Proprietary and Confidential-


©2014 himagine solutions inc. P a g e | 98
____________________________________________________________________________________________

____________________________________________________________________________________________

Scenario 2 (Coding):
Diagnosis: A patient is admitted to the hospital and taken to the ICU with a T 102, Pulse 120,
RR 25, altered mental status, BP 85/62, confusion and WBCs 25.2. He is treated with IV
antibiotics. The day following admission, patient has decreased urine output with a rise in
BUN/Creatinine. Patient is diagnosed with Acute Renal Failure. With appropriate
documentation of Severe Sepsis with Acute Renal Failure what are the codes to assign?

____________________________________________________________________________________________

____________________________________________________________________________________________

Scenario 3 (Coding):
Diagnosis: Patient is a 70 year old male admitted from a nursing home to ER with fever, WBC
25.4, shaking, chills, confusion and low BP 90/50. Patient’s blood cultures are positive for
streptococcus A and chest x-ray is positive for infiltrate. The patient also has a Stage 3
decubitus ulcer of the left hip demonstrating inflammation and has a serous discharge. With
appropriate documentation of Streptococcus A sepsis with pneumonia and Stage 3 decubitus
ulcer of left hip, what codes would be assigned?

____________________________________________________________________________________________

____________________________________________________________________________________________

-Proprietary and Confidential-


©2014 himagine solutions inc. P a g e | 99
Answer Key

-Proprietary and Confidential-


©2014 himagine solutions inc. P a g e | 100
Scenario Answers: ICD- 10-CM
Scenario 9
Infectious Disease
D57.01
Scenario 1
B37.3, Z16.19 Endocrine, Nutritional and
Metabolic Disease
Scenario 2
A41.50, R65.20, J96.00 Scenario 10
E11.22, N18.3, Z79.4
Instructions say to code A41.50 1st
Code E11.22 directs coder to use additional
Scenario 3 codes.
B20, B59
Scenario 11
Neoplasm E10.42, Z91.14, Z79.4

Scenario 4 Mental and Behavioral Disorders


Z51.11, C17.8, Z90.49
Scenario 12
Scenario 5 F10.229, Y90.1
C50.412, C77.3
Scenario 13
Scenario 6 F20.0, F60.3
Z51.11, C34.11
Scenario 14
Blood and Blood Forming Organs F10.239, R56.9
and Certain Disorders Involving
Diseases of the Nervous System
the Immune Mechanism
Scenario 15
Scenario 7 G40.B19
D57.01
Scenario 16
Scenario 8 G81.91
D53.9, K29.70
Scenario 17
A nonspecific code is used here due to lack
G30.9, F02.81
of specific documentation. The term
“probably” cannot be used as a definitive
Diseases of the Eye and Adnexa
code for lack of nutrition in outpatient
coding. Scenario 18

-Proprietary and Confidential-


©2014 himagine solutions inc. P a g e | 170
H01.001, H01.004 Scenario 28
J03.00
Since there is no code for non-ulcerative or
bilateral coding, two codes are needed and Scenario 29
unspecified is correct. J44.9

Scenario 19 See includes notes


H40.11X0, H35.30
Scenario 30
Diseases of the Ear and Mastoid J96.00

Scenario 20 Scenario 31
H65.04 J45.42, J44.1

Scenario 21 Diseases of the Digestive System


H80.03, H90.0, H95.31
Scenario 32
Surgical Misadventures are now found K40.31
within the chapter and should not be billed
to the patient. Scenario 33
K80.00
Scenario 22
I25.118 Scenario 34
K65.0, B96.20
Scenario 23
I28.8 Notes state to include infectious agent

Scenario 24 Scenario 35
I21.02 K40.20, R07.2, J44.9, M54.5, I10, Z53.09

Scenario 25 Disease of the Skin and


I63.442, I69.391, I69.351 Subcutaneous Tissue
Scenario 26 Scenario 36
I22.1, I21.3, I48.91 L40.50

Scenario 27 Scenario 37
I21.4, I48.91 L03.115

Diseases of the Respiratory Scenario 38


System L89.224

-Proprietary and Confidential-


©2014 himagine solutions inc. P a g e | 171
Scenario 39 Scenario 49
L02.612, I96 O03.4, O13.1

Diseases of the Musculoskeletal Scenario 50


System and Connective Tissue O26.12, O10.012

Scenario 40 Perinatal (Newborn) Period


M24.411
Scenario 51
Scenario 41 Z38.00, Q86.0
M23.200
Scenario 52
Scenario 42 P36.2
M84.421K
Congenital, Malformations,
Scenario 43 Deformations and Chromosomal
M08.071, M08.072 Abnormalities
Diseases of the Genitourinary Scenario 53
System Q37.4

Scenario 44 Scenario 54
N73.9 Q54.2

Scenario 45 Symptoms, Signs, and Abnormal


N39.0, B96.20 Laboratory Findings, NEC
Notes state to include code for infectious Scenario 55
agent
R40.2111, R40.2211, R40.2311, R40.2134,
R40.2234, R40.2344
Scenario 46
N30.11
Scenario 56
Scenario 47 R00.1
N18.3, Z94.0, E89.0, Z85.850
Injury, Poisoning and Certain
Pregnancy, Childbirth, and Other Consequences of External
Puerperium Causes

Scenario 48 Scenario 57
O63.1, Z3A.39, Z37.0 S22.41XA, S62.101A

-Proprietary and Confidential-


©2014 himagine solutions inc. P a g e | 172
Scenario 58 Scenario 62
I13.2,I50.9, N18.5, T50.1X6A, Z91.130 S06.9X2A, R40.2121, R40.2211, R40.2311,
Y04.0XXA, Y92.830, Y93.01, Y99.8
Scenario 59
S42.431D Factors Influencing Health Status
and Contact with Health Services
Scenario 60
S02.0XXA, S06.5X2A Scenario 63
Z02.0
External Causes of Comorbidity
Scenario 64
Scenario 61 Z44.121, Z89.511
S42.401K, V00.121D

-Proprietary and Confidential-


©2014 himagine solutions inc. P a g e | 173
Case Study Answers: ICD- Endocrine Disease
10 CM & ICD-10-PCS Scenario 1
Answer: E05.20, R00.2
Respiratory Disease
Scenario 2
Scenario 1 Answer: E66.01, Z68.39, 0DV64CZ
Answer: J96.01, J44.0, J20.0; 5A1945Z
Scenario 3
Scenario 2 Answer: E28.310
Answer: C18.7, C77.2, C78.7, J95.811;
0DTN0ZZ, 0FB00ZX, 07BB0ZZ, 0B9C30Z
Scenario 4
Answer: E11.649, Z79.4
Scenario 3
Answer: M05.641, M05.642, J84.17 (if
Scenario 5
Answer: E86.0, A02.0
caused by RA); if caused by Methotrexate it
would be coded J70.2, T45.1x5A
Blood and Blood Forming
Scenario 4
Scenario/Exercise 1:
Answer: J11.00, E86.0
Answer: B, D, A, E, F, C
Circulatory Disease Scenario 2
Scenario 1 Answer: O99.012, D50.9, 30233N1
Answer: I25.10 CAD, I49.5 SSS, I11.0 HCVD,
Scenario 3
I50.9 CHF, 0JH606Z (Pacemaker), 02H63JZ
Answer: D57.00
(A lead, right atrium), 02HK3JZ(a lead,
right ventricle), 027134Z (PTCA w/DES) Scenario 4
Answer: D73.1, D59.9, 07TP0ZZ
Scenario 2
Answer: I24.8, 02703DZ
Urinary/Kidney Disease
Scenario 3
Scenario 1
Answer: I50.23, I20.0
Answer: N00.6, 0TB13ZX
Scenario 4 Scenario 2
Answer: I71.4, I10, Z82.49, 04U00JZ
Answer: E11.22, N18.4, I12.9
Scenario 5 Scenario 3
Answer: I65.22, I69.190, 03CJ0ZZ
Answer: N13.2, BT221ZZ, BT120ZZ

-Proprietary and Confidential-


©2014 himagine solutions inc. P a g e | 177
Nervous System

Scenario 1
Answer: G30.1, F02.81, Z91.83

What you need to have documented to


code Alzheimer’s appropriately is: whether
it is early or late onset, senile or presenile,
with or without dementia and with our
without behavioral disturbances.

Scenario 2
Answer: G21.11, T43.3x5A

Scenario 3
Answer: I63.312, I69.320, I69.390

OB/GYN
Scenario 1
Answer: N82.2, 0DB80ZZ, 0UUG0JZ

Scenario 2
Answer: N98.0, N71.0

Scenario 3
Answer: N99.3, 0UBGXZZ, 0UUGXJZ

Infectious Disease
Scenario 1
Answer: No, there is no classification for
urosepsis. Patient has clinical signs of
sepsis, would require retrospective query.

Scenario 2
Answer: A41.9, R65.20, N17.9

Scenario 3
Answer: A40.0, J15.4, L89.22

-Proprietary and Confidential-


©2014 himagine solutions inc. P a g e | 178
Resources and References

-Proprietary and Confidential-


himagine solutions P a g e | 179
Resources and References
ICD-10-CPS The Complete Official Draft Code Set. 2014. Ingenix.

Pocket Guide of ICD-10-CM and ICD-10-PCS. Ann M. Zeisset, RHIT, CCS, CCS-P and Sue E. Bowman, RHIA,
CCS.

ICD-10-CM/PCS Coder Training Manual. Ann Barta, MSA. RHIA, Kathryn DeVault, RHIA, CCS; and Ann
Zeisset, RHIT, CCS, CCS-P
ICD-10-CM/PCS Coder Training Manual Instructor’s Edition. Ann Barta, MSA, RHIA; Kathryn DeVault,
RHIA, CCS; and Ann Zeisset, RHIT, CCS, CCS-P. AHIMA. 2009.
http://www.cms.hhs.gov/ICD10
http://training.seer.cancer.gov/lymphoma/anatomy/chains/index.html
http://www.ahima.org/ICD10/
http://msjensen.cehd.umn.edu/webanatomy/
http://msjensen.cehd.umn.edu/1135/med_term_activites/roots_list.html
http://www.cms.gov/Medicare/Coding/ICD10/2014-ICD-10-CM-and-GEMs.html
http://www.cms.gov/Medicare/Coding/ICD10/2014-ICD-10-PCS.html

Important ICD-10 Websites:

ICD-10-CM Website Information:


2013 and 2014 ICD-10-CM is available at http://www.cdc.gov/nchs/icd/icd10cm.htm or
http://www.cms.hhs.gov/ICD10
2013 and 2014 ICD-10-CM Index to Diseases and Injuries
2013 and 2014 ICD-10-CM Tabular List of Diseases and Injuries

Instructional Notations
2013 and 2014 Official Guidelines for Coding and Reporting
2013 and 2014 Table of Drugs and Chemicals
2013 and 2014 Neoplasm Table
2013 and 2014 Index to External Causes
2013 and 2014 Mapping ICD-9 to ICD-10

-Proprietary and Confidential-


himagine solutions P a g e | 180
Note Pages

-Proprietary and Confidential-


himagine solutions P a g e | 181
-Proprietary and Confidential-
himagine solutions P a g e | 182
-Proprietary and Confidential-
himagine solutions P a g e | 183
-Proprietary and Confidential-
himagine solutions P a g e | 184
-Proprietary and Confidential-
himagine solutions P a g e | 185

Anda mungkin juga menyukai